Sei sulla pagina 1di 78

CONTEDO

XXVII OLIMPADA BRASILEIRA DE MATEMTICA Problemas e Solues da Primeira Fase XXVII OLIMPADA BRASILEIRA DE MATEMTICA Problemas e Solues da Segunda Fase XXVII OLIMPADA BRASILEIRA DE MATEMTICA Problemas e Solues da Terceira Fase XXVII OLIMPADA BRASILEIRA DE MATEMTICA Problemas e Solues da Primeira Fase Nvel Universitrio XXVII OLIMPADA BRASILEIRA DE MATEMTICA Problemas e Solues da Segunda Fase Nvel Universitrio XXVII OLIMPADA BRASILEIRA DE MATEMTICA Premiados AGENDA OLMPICA COORDENADORES REGIONAIS

2 14

36

59

65

73 77 78

Sociedade Brasileira de Matemtica

XXVII OLIMPADA BRASILEIRA DE MATEMTICA


Problemas e Solues da Primeira Fase
PROBLEMAS NVEL 1 1. Sabendo-se que 9 174 532 13 = 119 268 916 , pode-se concluir que divisvel por 13 o nmero: A) 119 268 903 B) 119 268 907 C) 119 268 911 D) 119 268 913 E) 119 268 923 2. Numa caixa havia 3 meias vermelhas, 2 brancas e 1 preta. Professor Piraldo retirou 3 meias da caixa. Sabendo-se que nenhuma delas era preta, podemos afirmar sobre as 3 meias retiradas que: A) so da mesma cor. B) so vermelhas. B) uma vermelha e duas so brancas. D) uma branca e duas so vermelhas. E) pelo menos uma vermelha. 3. Diamantino colocou em um recipiente trs litros de gua e um litro de suco composto de 20% de polpa e 80% de gua. Depois de misturar tudo, que porcentagem do volume final polpa? A) 5% B) 7% C) 8% D) 20% E) 60% 4. Perguntado, Arnaldo diz que 1 bilho o mesmo que um milho de milhes. Professor Piraldo o corrigiu e disse que 1 bilho o mesmo que mil milhes. Qual a diferena entre essas duas respostas? A) 1 000 B) 999 000 C) 1 000 000 D) 999 000 000 E) 999 000 000 000 5. Numa seqncia, cada termo, a partir do terceiro, a soma dos dois termos anteriores mais prximos. O segundo termo igual a 1 e o quinto termo vale 2005. Qual o sexto termo? A) 3 002 B) 3 008 C) 3 010 D) 4 002 E) 5 004

EUREKA! N24, 2006

Sociedade Brasileira de Matemtica

6. Um galo de mel fornece energia suficiente para uma abelha voar 7 milhes de quilmetros. Quantas abelhas iguais a ela conseguiriam voar mil quilmetros se houvesse 10 gales de mel para serem compartilhados entre elas? A) 7 000 B) 70 000 C) 700 000 D) 7 000 000 E) 70 000 000 7. Trs anos atrs, a populao de Pirajussara era igual populao que Tucupira tem hoje. De l para c, a populao de Pirajussara no mudou mas a populao de Tucupira cresceu 50%. Atualmente, as duas cidades somam 9000 habitantes. H trs anos, qual era a soma das duas populaes? A) 3 600 B) 4 500 C) 5 000 D) 6 000 E) 7 500 8. Um agricultor esperava receber cerca de 100 mil reais pela venda de sua safra. Entretanto, a falta de chuva provocou uma perda da safra avaliada entre 1 1 e do total previsto. Qual dos valores a seguir pode representar a perda do 5 4 agricultor? A) R$ 21.987,53 B) R$ 34.900,00 C) R$ 44.999,99 D) R$ 51.987,53 E) R$ 60.000,00 9. Devido a um defeito de impresso, um livro de 600 pginas apresenta em branco todas as pginas cujos nmeros so mltiplos de 3 ou de 4. Quantas pginas esto impressas? A) 100 B) 150 C) 250 D) 300 E) 430 10. Seis retngulos idnticos so reunidos para formar um retngulo maior conforme indicado na figura. Qual a rea deste retngulo maior? B) 280 cm2 A) 210 cm2 2 C) 430 cm D) 504 cm2 2 E) 588 cm

21 cm

11. O relgio do professor Piraldo, embora preciso, diferente, pois seus ponteiros se movem no sentido anti-horrio. Se voc olhar no espelho o relgio quando ele estiver marcando 2h23min, qual das seguintes imagens voc ver?

EUREKA! N24, 2006

Sociedade Brasileira de Matemtica

E
A) B)

E
C)

E
D)

E
E)

12. Uma placa decorativa consiste num quadrado de 4 metros de lado, pintada de forma simtrica com algumas faixas, conforme indicaes no desenho ao lado. Qual a frao da rea da placa que foi pintada?

1m 1m 1m 1m 1m

A)

1 2

B)

1 3

C)

3 8

D)

6 13

E)

7 11

1m

13. Pelculas de insulfilm so utilizadas em janelas de edifcios e vidros de veculos para reduzir a radiao solar. As pelculas so classificadas de acordo com seu grau de transparncia, ou seja, com o percentual da radiao solar que ela deixa passar. Colocando-se uma pelcula de 70% de transparncia sobre um vidro com 90% de transparncia, obtm-se uma reduo de radiao solar igual a : A) 3% B) 37% C) 40% D) 63% E) 160% 14. Na figura, os dois tringulos so eqilteros. Qual o valor do ngulo x?
x

75

65

A) 30o

B) 40o

C) 50o

D) 60o
10 10 5 5

E) 70o

15. Um serralheiro solda varetas de metal para produzir peas iguais que sero juntadas para formar o painel abaixo. O desenho ao lado apresenta as medidas, em centmetros, de uma dessas peas. O serralheiro usa exatamente 20
EUREKA! N24, 2006

5 10

Sociedade Brasileira de Matemtica

metros de vareta para fazer o seu trabalho.

Qual dos desenhos abaixo representa o final do painel?

A)

B)

C)

D)

E)

16. Dentre os nmeros 1, 2, 3, 4, 5, 6, 7, 8, 9 e 10, escolha alguns e coloque-os nos crculos brancos de tal forma que a soma dos nmeros em dois crculos vizinhos seja sempre um quadrado perfeito. Ateno: o 2 j foi colocado em um dos crculos e no permitido colocar nmeros repetidos; alm disso, crculos separados pelo retngulo preto no so vizinhos.
2

A soma dos nmeros colocados em todos os crculos brancos : A) 36 B) 46 C) 47 D) 49 E) 55 17. Figuras com mesma forma representam objetos de mesma massa. Quantos quadrados so necessrios para que a ltima balana fique em equilbrio?

A) 7
EUREKA! N24, 2006

B) 8

C) 9

D) 10

E) 12

Sociedade Brasileira de Matemtica

18. As 10 cadeiras de uma mesa circular foram numeradas com nmeros consecutivos de dois algarismos, entre os quais h dois que so quadrados perfeitos. Carlos sentou-se na cadeira com o maior nmero e Janana, sua namorada, sentou-se na cadeira com o menor nmero. Qual a soma dos nmeros dessas duas cadeiras? A) 29 B) 36 C) 37 D) 41 E) 64 19. Em um ano, no mximo quantos meses tm cinco domingos? A) 3 B) 4 C) 5 D) 6

E) 7

20. As nove casas de um tabuleiro 3 3 devem ser pintadas de foram que cada coluna, cada linha e cada uma das duas diagonais no tenham duas casas de mesma cor. Qual o menor nmero de cores necessrias para isso? A) 3 B) 4 C) 5 D) 6 E) 7

PROBLEMAS NVEL 2 1. Uma loja de sabonetes realiza uma promoo com o anncio "Compre um e leve outro pela metade do preo. Outra promoo que a loja poderia fazer oferecendo o mesmo desconto percentual A) "Leve dois e pague um B) "Leve trs e pague um C) "Leve trs e pague dois D) "Leve quatro e pague trs E) "Leve cinco e pague quatro 2. 3. 4. 5. Veja o problema No. 13 do Nvel 1. Veja o problema No. 10 do Nvel 1. Veja o problema No. 4 do Nvel 1. Veja o problema No. 9 do Nvel 1.

6. Platina um metal muito raro, mais raro at do que ouro. Sua densidade 21,45 g/cm3. Suponha que a produo mundial de platina foi de cerca de 110 toneladas em cada um dos ltimos 50 anos e desprezvel antes disso. Assinale a alternativa com o objeto cujo volume mais prximo do volume de platina produzido no mundo em toda a histria. A) uma caixa de sapatos B) uma piscina C) um edifcio de dez andares D) o monte Pascoal E) a Lua 7. Veja o problema No. 5 do Nvel 1. 8. Veja o problema No. 17 do Nvel 1.
EUREKA! N24, 2006

Sociedade Brasileira de Matemtica

9. Entre treze reais no nulos h mais nmeros positivos do que negativos. Dentre os
13 12 = 78 produtos de dois 2

dos treze nmeros, 22 so

negativos. Quantos nmeros dentre os treze nmeros dados so negativos? A) 2 B) 7 C) 8 D) 9 E) 10 10. O desenho ao lado mostra um pedao de papelo que ser dobrado e colado nas bordas para formar uma caixa retangular. Os ngulos nos cantos do papelo so todos retos. Qual ser o volume da caixa em cm3?
15 cm

40 cm

A) 1 500 E) 12 000

B) 3 000

C) 4 500

20 cm

D) 6 000

11. Sendo a, b e c nmeros reais, pela propriedade distributiva da multiplicao em relao adio, verdade que a (b + c) = (a b) + (a c). A distributiva da adio em relao multiplicao a + (b c) = (a + b) (a + c) no sempre verdadeira, mas ocorre se, e somente se, A) a = b = c = 1 ou a = 0
3

B) a = b = c D) a + b + c = 1 ou a = 0

C) A igualdade nunca ocorre E) a = b = c = 0

12. Em certa cidade, acontece um fato interessante. Dez por cento dos Baianos dizem que so Paulistas e dez por cento dos Paulistas dizem que so Baianos. Todos os outros Paulistas e Baianos assumem a sua verdadeira origem. Dentre os Paulistas e Baianos, 20% dizem que so Paulistas. Que percentual os realmente Paulistas representam dentre os Paulistas e Baianos? A) 12,5% B) 18% C) 20% D) 22% E) 22,5% 13. Veja o problema No. 14 do Nvel 1.
EUREKA! N24, 2006

Sociedade Brasileira de Matemtica

14. As letras O, B e M representam nmeros inteiros. Se O B M = 240, O B + M = 46 e O + B M = 64, quanto vale O + B + M? A) 19 B) 20 C) 21 D) 24 E) 36 15. Veja o problema No. 15 do Nvel 1. 16. Veja o problema No. 19 do Nvel 1. 17. Quantos nmeros entre 10 e 13000, quando lidos da esquerda para a direita, so formados por dgitos consecutivos e em ordem crescente? Exemplificando, 456 um desses nmeros, mas 7890 no : A) 10 B) 13 C) 18 D) 22 E) 25 18. Um piloto percorreu trs trechos de um rali, de extenses 240 km, 300 km e 400 km, respectivamente. As velocidades mdias nos trs trechos foram 40 km/h, 75 km/h e 80 km/h, mas no necessariamente nessa ordem. Podemos garantir que o tempo total em horas gasto pelo piloto nos trs trechos : A) menor ou igual a 13 horas B) maior ou igual a 13 horas e menor ou igual a 16 horas C) maior ou igual a 14 horas e menor ou igual a 17 horas D) maior ou igual a 15 horas e menor ou igual a 18 horas E) maior ou igual a 18 horas 19. Na figura, todas as circunferncias menores tm o mesmo raio r e os centros das circunferncias que tocam a circunferncia maior so vrtices de um a quadrado. Sejam a e b as reas cinzas indicadas na figura. Ento a razo b igual a:

b a

A) 1
2

B) 2
3

C) 1

D) 3
2

E) 2

EUREKA! N24, 2006

Sociedade Brasileira de Matemtica

20. Um professor de Ingls d aula particular para uma classe de 9 alunos, dos quais pelo menos um brasileiro. Se o professor escolher 4 alunos para fazer uma apresentao, ter no grupo pelo menos dois alunos de mesma nacionalidade; se escolher 5 alunos, ter no mximo trs alunos de mesma nacionalidade. Quantos brasileiros existem na classe? A) 1 B) 2 C) 3 D) 4 E) 5

21. Um relgio, com ponteiros de horas, minutos e segundos, faz plim toda vez que um ponteiro ultrapassa outro no mostrador. O nmero de plins registrados em um certo dia, no perodo entre as 12 horas e 1 segundo e as 23 horas, 59 minutos e 59 segundos : A) 732 B) 1438 C) 1440 D) 1446 E) 1452

22. Na figura, a reta PQ toca em N o crculo que passa por L, M e N. A reta LM corta a reta PQ em R. Se LM = LN e a medida do ngulo PNL , < 60o, quanto mede o ngulo LRP?
L

P N R Q

A)3 180o

B)180o 2

C) 180o

D) 90o /2

E)

23. Os inteiros positivos x e y satisfazem a equao

x+ 1 y x 1 y =1. 2 2
Qual das alternativas apresenta um possvel valor de y? A) 5 B) 6 C) 7 D) 8 E) 9

24. Veja o problema No. 16 do Nvel 1.

EUREKA! N24, 2006

Sociedade Brasileira de Matemtica

25. Um bloco de dimenses 1 2 3 colocado sobre um tabuleiro 8 8, como mostra a figura, com a face X, de dimenses 1 2, virada para baixo. Giramos o bloco em torno de uma de suas arestas de modo que a face Y fique virada para baixo. Em seguida, giramos novamente o bloco, mas desta vez de modo que a face Z fique virada para baixo. Giramos o bloco mais trs vezes, fazendo com que as faces X, Y e Z fiquem viradas para baixo, nessa ordem. Quantos quadradinhos diferentes do tabuleiro estiveram em contato com o bloco?

Z Y

A) 18 PROBLEMAS NVEL 3

B) 19

C) 20

D) 21

E)22

1. Veja o problema No. 17 do Nvel 2. 2. Os pontos L, M e N so pontos mdios de arestas do cubo, como mostra a figura. Quanto mede o ngulo LMN?
N

A) 90

B) 105

C) 120o

D) 135o

E) 150o

3. Veja o problema No. 22 do Nvel 2. 4. Veja o problema No. 14 do Nvel 2.


EUREKA! N24, 2006

10

Sociedade Brasileira de Matemtica

5. Esmeralda digitou corretamente um mltiplo de 7 muito grande, com 4010 algarismos. Da esquerda para a direita, os seus algarismos so 2004 algarismos 1, um algarismo n e 2005 algarismos 2. Qual o valor de n? A) 3 B) 4 C) 5 D) 6 E) 7 6. 7. 8. 9. Veja o problema No. 23 do Nvel 2. Veja o problema No. 25 do Nvel 2. Veja o problema No. 1 do Nvel 2. Veja o problema No. 6 do Nvel 2.

10. A figura mostra um cubo de aresta 1 no qual todas as doze diagonais de face foram desenhadas. Com isso, criou-se uma rede com 14 vrtices (os 8 vrtices do cubo e os 6 centros de faces) e 36 arestas (as 12 arestas do cubo e mais 4 sobre cada uma das 6 faces). Qual o comprimento do menor caminho que formado por arestas da rede e que passa por todos os 14 vrtices?

A) 1 + 6 2 E) 12 + 12 2

B) 4 + 2 2

C) 6

D) 8 + 6 2

11. Uma das faces de um poliedro um hexgono regular. Qual a quantidade mnima de arestas que esse poliedro pode ter? A) 7 B) 9 C) 12 D) 15 E) 18 12. Veja o problema No. 19 do Nvel 1. 13. O ponto D pertence ao lado BC do tringulo ABC. Sabendo que AB = AD = 2, ngulos BAD e CAD so congruentes, ento a medida do BD = 1 e os segmento CD : 3 4 5 6 7 A) B) C) D) E) 2 3 4 5 6

EUREKA! N24, 2006

11

Sociedade Brasileira de Matemtica

14. Esmeralda adora os nmeros triangulares (ou seja, os nmeros 1, 3, 6, 10, 15, 21, 28), tanto que mudou de lugar os nmeros 1, 2, 3, , 11 do relgio de parede do seu quarto de modo que a soma de cada par de nmeros vizinhos um nmero triangular. Ela deixou o 12 no seu lugar original. Que nmero ocupa o lugar que era do 6 no relgio original? A) 1 B) 4 C) 5 D) 10 E) 11 15. Os termos an de uma seqncia de inteiros positivos satisfazem a relao an+3 = an+2(an+1 + an) para n = 1, 2, 3 Se a5 = 35, quanto a4? A) 1 B) 3 C) 5 D) 7 E) 9 16. Veja o problema No. 11 do Nvel 2. 17. Veja o problema No. 19 do Nvel 2. 18. Entre treze no nulos h mais nmeros positivos do que 13 12 negativos. Dentre os = 78 produtos de dois dos treze nmeros, 22 2 so negativos. Quantos nmeros dentre os treze nmeros dados so negativos? A) 2 B) 7 C) 8 D) 9 E) 10 reais

19. Traando as quatro retas perpendiculares aos lados de um paralelogramo no retngulo pelos seus pontos mdios, obtm-se uma regio do plano limitada por essas quatro retas. Podemos afirmar que a rea dessa regio igual rea do paralelogramo se um dos ngulos do paralelogramo for igual a: A) 30o B) 45o C) 60o D) 75o E) 90o 20. O nmero (2 + 2)3 (3 2) 4 + (2 2)3 (3 + 2) 4 : A) inteiro mpar B) inteiro par C) racional no inteiro D) irracional positivo irracional negativo 21. Sejam A = 10(log10 2005) , B = 20053 e C = 2 A) A < B < C B) A < C < B C) B < A < C D) B < C < A
2

E)

2005

. Ento: E) C < A < B

22. Veja o problema No. 18 do Nvel 2.

EUREKA! N24, 2006

12

Sociedade Brasileira de Matemtica

23. Dois nmeros inteiros so chamados de primanos quando pertencem a uma progresso aritmtica de nmeros primos com pelo menos trs termos. Por exemplo, os nmeros 41 e 59 so primanos pois pertencem progresso aritmtica (41; 47; 53; 59) que contm somente nmeros primos. Assinale a alternativa com dois nmeros que no so primanos. A) 7 e 11 B) 13 e 53 C) 41 e 131 D) 31 e 43 E) 23 e 41 24. Um relgio, com ponteiros de horas, minutos e segundos, faz plim toda vez que um ponteiro ultrapassa outro no mostrador. O nmero de plins registrados em um certo dia no perodo entre as 12 horas e 1 segundo e as 23 horas, 59 minutos e 59 segundos : A) 732 B) 1438 C) 1440 D) 1446 E) 1452 25. Veja o problema No. 20 do Nvel 2.

GABARITO
NVEL 1 (5a. e 6a. sries) 1) A 6) B 2) E 7) E 3) A 8) A 4) E 9) D 5) B 10) E 11) A 12) C 13) B 14) B 15) B 16) B 17) D 18) D 19) C 20) C

NVEL 2 (7a. e 8a. sries) 1) D 6) B 2) B 7) B 3) E 8) D 4) E 9) A 5) D 10) B NVEL 3 (Ensino Mdio) 1) D 6) C 2) C 7) B 3) Anulada 8) D 4) B 9) B 5) B 10) A

11) D 12) A 13) B 14) B 15) B

16) C 17) D 18) Anulada 19) C 20) C

21) Anulada 22) Anulada 23) C 24) B 25) B

11) C 12) C 13) B 14) C 15) D

16) D 17) C 18) A 19) B 20) B

21) C 22) Anulada 23) B 24) Anulada 25) C

EUREKA! N24, 2006

13

Sociedade Brasileira de Matemtica

XXVII OLIMPADA BRASILEIRA DE MATEMTICA


Problemas e Solues da Segunda Fase PROBLEMAS Nvel 1 PARTE A (Cada problema vale 5 pontos)
01. O tanque do carro de Esmeralda, com capacidade de 60 litros, contm uma

mistura de 20% de lcool e 80% de gasolina ocupando metade de sua capacidade. Esmeralda pediu para colocar lcool no tanque at que a mistura ficasse com quantidades iguais de lcool e gasolina. Quantos litros de lcool devem ser colocados?
02. Na seqncia de nmeros 1, a, 2, b, c, d, ... dizemos que o primeiro termo 1, o

segundo termo a, o terceiro termo 2, o quarto termo b, e assim por diante. Sabe-se que esta seqncia tem 2005 termos e que cada termo, a partir do terceiro, a mdia aritmtica de todos os termos anteriores. Qual o ltimo termo dessa seqncia?
03. Natasha supersticiosa e, ao numerar as 200 pginas de seu dirio, comeou do 1 mas pulou todos os nmeros nos quais os algarismos 1 e 3 aparecem juntos, em qualquer ordem. Por exemplo, os nmeros 31 e 137 no aparecem no dirio, porm 103 aparece. Qual foi o nmero que Natasha escreveu na ltima pgina do seu dirio? 04. Juliana foi escrevendo os nmeros inteiros positivos em quadrados de papelo,

colados lado a lado por fitas adesivas representadas pelos retngulos escuros no desenho abaixo. Note que cada fila de quadrados tem um quadrado a mais que a fila de cima. Ela escreveu at o nmero 105 e parou. Quantos pedaos de fita adesiva ela usou?
1 2 4 7 3 5 8 6 9 10

05. Lara tem cubos iguais e quer pint-los de maneiras diferentes, utilizando as cores laranja ou azul para colorir cada uma de suas faces.
EUREKA! N24, 2006

14

Sociedade Brasileira de Matemtica

Para que dois cubos no se confundam, no deve ser possvel girar um deles de forma que fique idntico ao outro. Por exemplo, h uma nica maneira de pintar o cubo com uma face laranja e cinco azuis. Quantos cubos pintados de modos diferentes ela consegue obter?
06. Um carpinteiro fabrica caixas de madeira abertas na parte de cima, pregando duas placas retangulares de 600 cm2 cada uma, duas placas retangulares de 1200 cm2 cada uma e uma placa retangular de 800 cm2, conforme representado no desenho. Qual o volume, em litros, da caixa? Note que l litro = 1000 cm3.

PROBLEMAS Nvel 1 PARTE B (Cada problema vale 10 pontos)


PROBLEMA 1

Quatro peas iguais, em forma de tringulo retngulo, foram dispostas de dois modos diferentes, como mostram as figuras. H I D A L B K C E P J M N G

F Os quadrados ABCD e EFGH tm lados respectivamente iguais a 3 cm e 9 cm. Calcule as reas dos quadrados IJKL e MNOP.

EUREKA! N24, 2006

15

Sociedade Brasileira de Matemtica

PROBLEMA 2

Considere trs nmeros inteiros positivos consecutivos de trs algarismos tais que o menor mltiplo de 7, o seguinte mltiplo de 9 e o maior mltiplo de 11. Escreva todas as seqncias de nmeros que satisfazem essas propriedades.
PROBLEMA 3

Cada pea de um jogo de domin possui duas casas numeradas. Considere as 6 peas formadas apenas pelos nmeros 1, 2 e 3. (a) De quantos modos possvel colocar todas estas peas alinhadas em seqncia, de modo que o nmero da casa da direita de cada pea seja igual ao nmero da casa da esquerda da pea imediatamente direita? A seguir, mostramos dois exemplos:

(b) Explique por que no possvel fazer o mesmo com todas as 10 peas formadas apenas pelos nmeros 1, 2, 3 e 4. PROBLEMAS Nvel 2 PARTE A (Cada problema vale 4 pontos)
01. Veja o problema No. 3 do Nvel 1 Parte A. 02. Quatro peas iguais, em forma de tringulo retngulo, foram dispostas de dois modos diferentes, como mostram as figuras abaixo.

H I D A B L K E P F O C J M N G

EUREKA! N24, 2006

16

Sociedade Brasileira de Matemtica

Os quadrados ABCD e EFGH tm lados respectivamente iguais a 3 cm e 9 cm. Determine a medida do lado do quadrado IJKL.
03. Veja o problema No. 4 do Nvel 1 parte A. 04. Um terreno quadrangular foi dividido em quatro lotes menores por duas cercas retas unindo os pontos mdios dos lados do terreno. As reas de trs dos lotes esto indicadas em metros quadrados no mapa a seguir.

250

200

210

Qual a rea do quarto lote, representado pela regio escura no mapa?


05. Seja a um nmero inteiro positivo tal que a mltiplo de 5, a + 1 mltiplo de

7, a + 2 mltiplo de 9 e a + 3 mltiplo de 11. Determine o menor valor que a pode assumir. PROBLEMAS Nvel 2 PARTE B (Cada problema vale 10 pontos)
PROBLEMA 1

Gabriel resolveu uma prova de matemtica com questes de lgebra, geometria e lgica. Aps checar o resultado da prova Gabriel observou que respondeu corretamente 50% das questes de lgebra, 70% das questes de geometria e 80% das questes de lgica. Gabriel observou, tambm, que respondeu corretamente 62% das questes de lgebra e lgica e 74% das questes de geometria e lgica. Qual a porcentagem de questes corretas da prova de Gabriel?
PROBLEMA 2

O canto de um quadrado de cartolina foi cortado com uma tesoura. A soma dos comprimentos dos catetos do tringulo recortado igual ao comprimento do lado do quadrado. Qual o valor da soma dos ngulos e marcados na figura abaixo?

EUREKA! N24, 2006

17

Sociedade Brasileira de Matemtica

27

PROBLEMA 3

(a) Fatore a expresso x 2 9 xy + 8 y 2 . (b) Determine todos os pares de inteiros (x; y) tais que 9 xy x 2 8 y 2 = 2005 .
PROBLEMA 4

Veja o problema No. 3 do Nvel 1 Parte B.

PROBLEMAS Nvel 3 PARTE A (Cada problema vale 4 pontos)

01. Na figura, ABCDE um pentgono regular e AEF um tringulo eqiltero. Seja P um ponto sobre o segmento BF , no interior de ABCDE, e tal que o ngulo A mede 12, como mostra a figura abaixo. PE
F A P B E

Calcule a medida, em graus, do ngulo PC.

EUREKA! N24, 2006

18

Sociedade Brasileira de Matemtica

02. Seja a um nmero inteiro positivo tal que a mltiplo de 5, a + 1 mltiplo de

7, a + 2 mltiplo de 9 e a + 3 mltiplo de 11. Determine o menor valor que a pode assumir.


03. Veja o problema No. 4 do Nvel 2 parte A. 04. A funo f : \ \

satisfaz f ( x + f ( y )) = x + f ( f ( y )) para todos os nmeros reais x e y. Sabendo que f (2) = 8 , calcule f(2005).

05. Voc tem que determinar o polinmio p(x) de coeficientes inteiros positivos

fazendo perguntas da forma Qual o valor numrico de p(k)?, sendo k um inteiro positivo sua escolha. Qual o menor nmero de perguntas suficiente para garantir que se descubra o polinmio? PROBLEMAS Nvel 3 PARTE B (Cada problema vale 10 pontos)
PROBLEMA 1

Determine todos os pares de inteiros (x; y) tais que 9 xy x 2 8 y 2 = 2005 .


PROBLEMA 2

Um prisma reto e tem como base um tringulo equiltero. Um plano corta o prisma mas no corta nenhuma de suas bases, determinando uma seco triangular de lados a, b e c. Calcule o lado da base do prisma em funo de a, b e c.
PROBLEMA 3

No campeonato tumboliano de futebol, cada vitria vale trs pontos, cada empate vale um ponto e cada derrota vale zero ponto. Um resultado uma vitria, empate ou derrota. Sabe-se que o Flameiras no sofreu nenhuma derrota e tem 20 pontos, mas no se sabe quantas partidas esse time jogou. Quantas seqncias ordenadas de resultados o Flameiras pode ter obtido? Representando vitria por V, empate por E e derrota por D, duas possibilidades, por exemplo, so (V, E, E, V, E, V, V, V, E, E) e (E, V, V, V, V, V, E, V).
PROBLEMA 4

Determine o menor valor possvel do maior termo de uma progresso aritmtica com todos os seus sete termos a1, a2, a3, a4, a5, a6, a7 primos positivos distintos.
EUREKA! N24, 2006

19

Sociedade Brasileira de Matemtica

Curiosidade: No ano passado, os ex-olmpicos Terence Tao (Austrlia, ouro na IMO 1988) e Ben Green (Reino Unido, prata na IMO 1994) provaram que existem progresses aritmticas arbitrariamente grandes com todos os termos primos positivos. Tal questo remonta ao sculo XVIII, aparecendo nas pesquisas de Lagrange e Waring. Solues Nvel 1 Segunda Fase Parte A Problema Resposta 01 18 02 2 03 214 04 182 05 10 06 24

01. O tanque contm uma mistura de 30 litros, sendo 0, 2 30 = 6 litros de lcool e 30 6 = 24 litros de gasolina. Portanto, para que as quantidades de gasolina e lcool fiquem iguais, devem ser colocados no tanque 24 6 = 18 litros de lcool. 02. Como 2 a mdia aritmtica de 1 e a, podemos escrever

1+ a = 4 a = 3; d=

portanto,

b=

1+ 3 + 2 + 2 + 2 = 2 . Esses exemplos sugerem que todos os termos, a 5

1+ 2 + 3 = 2; 3

1+ a = 2 , logo 2 1+ 3 + 2 + 2 = 2; c= 4

partir do terceiro, so iguais a 2. De fato, quando introduzimos em uma seqncia um termo igual mdia de todos os termos da seqncia, a mdia da nova seqncia a mesma que a da seqncia anterior. Assim, o ltimo termo da seqncia dada 2.
03. Natasha pulou os nmeros 13, 31, 113, 130,131, 132, ..., 139, num total de 13

nmeros. Portanto, na ltima pgina do seu dirio escreveu o nmero 200 + 13 +1 = 214. 04. Olhando para o ltimo nmero da fila n, vemos que ele a soma de todos os nmeros de 1 a n: por exemplo, na fila 4, o ltimo nmero da fila 1 + 2 + 3 + 4 = 10. Note que para obter a quantidade de nmeros at uma certa fila, basta somar o nmero da fila ao total de nmeros que havia antes dessa fila. Assim, temos, fila 5 : 15, fila 6: 21, fila 7: 28, fila 8: 36, fila 9: 45, fila 10: 55, fila 11: 66, fila 12: 78, fila 13: 91, fila 14: 105
EUREKA! N24, 2006

20

Sociedade Brasileira de Matemtica

O nmero de fitas adesivas horizontais entre uma fila n 1 e uma fila n igual a n 1 e o nmero de fitas adesivas verticais numa fila n igual n 1. Portanto, at a fila nmero 14, o nmero de fitas

(1 + 2 +  + 13) + (1 + 2 +  + 13 ) = 2

13 14 = 182. 2

05. Todas as faces azuis: uma maneira. Cinco faces azuis e uma amarela: uma maneira. Quatro faces azuis e duas amarelas: duas maneiras (duas faces amarelas opostas ou duas faces amarelas adjacentes). Trs faces azuis e trs faces amarelas: duas maneiras (trs azuis com um vrtice comum uma maneira ou trs azuis com uma aresta comum duas a duas uma maneira) Duas faces azuis e quatro amarelas: duas maneiras Uma face azul e cinco amarelas: uma maneira. Todas as faces amarelas: uma maneira. Portanto, o nmero de maneiras diferentes de pintar o cubo 10. 06. Sejam a, b e c as medidas da caixa, conforme indicado no desenho ao lado. Segundo o enunciado, podemos escrever ab = 600, ac = 1200 e bc = 800. Sabemos que o volume da caixa abc. Utilizando as propriedades das igualdades e de potncias, podemos escrever

( ab ) ( ac ) (bc ) = 600 1200 800 a 2 b2 c2 = 2 3 102 22 3 102 23 102 2 ( abc ) = 26 32 106 abc = 26 32 106 abc = 23 3 103 = 24 1000 cm3

Como 1 litro igual a 1000 cm3, conclumos que o volume da caixa de 24 litros. Solues Nvel 1 Segunda Fase Parte B
SOLUO DO PROBLEMA 1: 1 maneira: O quadrado IJKL e o quadrado MNOP tm como lados as hipotenusas

dos tringulos retngulos dados, logo tm a mesma rea s. Fazendo os dois quadrados coincidirem, conclumos que o dobro da soma t das reas dos quatro tringulos retngulos a diferena entre as reas dos quadrados IJKL e EFGH, ou seja, 2t 92 32 2t 72 t 36 . Assim, s = 9 + 36 = 81 36 = 45 cm2.

   

2 maneira: No quadrado IJKL, seja JC = x. Ento IC = ID + DC = JC + DC = x + 3. Ento, no quadrado EFGH, temos . Portanto, a rea do quadrado HN NG x 3 x 9 2 x 6 x 3

   
21

EUREKA! N24, 2006

Sociedade Brasileira de Matemtica

IJKL, igual soma das reas dos quatro tringulos retngulos com a rea do quadrado ABCD, vale 4

3 32 3
3  36 9  45 e
2

a rea do quadrado

MNOP, igual diferena entre a rea do quadrado EFGH e a soma das reas dos quatro tringulos retngulos, vale 92
SOLUO DO PROBLEMA 2:

 4 3 3 3
 81 36  45 cm .
2

Seja n = abc mltiplo de 11; ento n 1 deve ser mltiplo de 9 e n 2 deve ser mltiplo de 7. Seja c 0 : Como abc mltiplo de 11, podemos ter a b c 0 ou a b c 11 . Como abc 1 mltiplo de 9, podemos ter a b c 1 9 ou a b c 1 18 . No caso de a + b + c 1 = 0 , teramos n 1 = 99 n = 100 , que no mltiplo de 11. Assim, simultaneamente, somente podemos

 

 

 

 

ter (i )

a + b + c = 10 2b = 10 b=5 a+c =b a+c=b a+c=5

ou

(ii )

a + b + c = 19 2b + 11 = 19 b=4 a + c = b + 11 a + c = b + 11 a + c = 15

No caso (i) existem as seguintes possibilidades para n: 154, 253, 352, 451, que so mltiplos de 11; para n 1 temos os nmeros 153, 252, 351, 450 e 549 so mltiplos de 9. Para os nmeros n 2 temos 152, 251, 350, 449 e 548, dos quais apenas 350 mltiplo de 7. No caso (ii) existem as seguintes possibilidades para n: 649, 748, 847 e 946, que so mltiplos de 11; para n 1 temos os nmeros 648, 747, 846 e 945 so mltiplos de 9. Para os nmeros n 2 temos 647, 746, 845 e 944, dos quais nenhum mltiplo de 7. Seja c = 0: Neste caso, n 1 tem os algarismos a, b 1 e 9. Assim, a + b 1 + 9 = 9 ou a + b 1 + 9 = 18 ou seja, a + b = 1 ou a + b = 10 . Como a b c a b 0 ou a b c a b 11 , conclumos que a = b. Assim, a = b = 5, o que fornece os nmeros n = 550, n 1 = 549 e n 2 = 548, que no divisvel por 7. Portanto, a nica seqncia de trs nmeros inteiros consecutivos nas condies dadas 350, 351 e 352.

   

   

EUREKA! N24, 2006

22

Sociedade Brasileira de Matemtica

SOLUO DO PROBLEMA 3: 1a maneira:

a) Podemos representar uma seqncia vlida como uma seqncia de pares ordenados. O primeiro exemplo a seqncia [(1,1),(1,2),(2,2),(2,3),(3,3),(3,1)] e, a partir dela, podemos criar outras seqncias vlidas movendo o par da esquerda para a direita (ou da direita para a esquerda). Assim, so vlidas as seqncias [(1,2),(2,2),(2,3),(3,3),(3,1),(1,1)], [(2,2),(2,3),(3,3),(3,1),(1,1), (1,2)],etc. num total de 6 seqncias diferentes. Mudando a posio dos nmeros dos pares ordenados, podemos criar outras 6 seqncias: [(2,1), (1,1), (1,3), (3,3),(3,2),(2,2)], [ (1,1), (1,3), (3,3),(3,2),(2,2), (2,1)], etc. Portanto, de acordo com as regras dadas h 12 modos de colocar as peas em seqncia.
2a maneira:

a) As pontas devem ter o mesmo nmero, pois eles aparecem um nmero par de vezes (se aparecer um nmero numa ponta e outro na outra, ento h pelo menos dois nmeros que aparecem um nmero mpar de vezes, o que no ocorre). Alguma pea com dois nmeros iguais deve aparecer em uma das pontas, pois do contrrio teramos trs das quatro peas centrais com duas iguais, vizinhas, o que impossvel). Sendo assim, a seqncia pode ser representada por XX-XY-YY-YZ-ZZ-ZX, onde para X temos trs possibilidades, para Y temos duas possibilidade e para Z, uma possibilidade, num total de 3.2.1 = 6 possibilidades para a seqncia que comea com uma dupla. Se a seqncia terminar com uma dupla, teremos novamente 6 possibilidades. Portanto, h 12 modos de colocar as seis peas em seqncia. b) Para cada nmero, existem 4 peas. Por exemplo, as peas com o nmero 1 esto desenhadas ao lado. O nmero de vezes em que aparece o nmero 1 mpar, logo a seqncia deveria comear com 1 e terminar com outro nmero ou comear com outro nmero e terminar com 1. Neste caso, os outros dois nmeros deveriam aparecer um nmero par de vezes, pois no estariam na ponta, mas isso no ocorre: todos os quatro nmeros aparecem um nmero mpar de vezes.

EUREKA! N24, 2006

23

Sociedade Brasileira de Matemtica

Solues Nvel 2 Segunda Fase Parte A Problema Resposta 01 214 02 -------03 182 04 240 05 1735

01. Natasha pulou os nmeros 13, 31, 113, 130,131, 132, ..., 139, num total de 13 nmeros. Portanto, na ltima pgina do seu dirio escreveu o nmero 200 + 13 +1 = 214. 02. Sejam x e y o maior e o menor catetos, respectivamente, do tringulo retngulo. Como o lado do quadrado ABCD mede 3 cm, temos x y = 3. Por outro lado, como o lado de EFGH mede 9 cm, temos x + y = 9. Resolvendo o sistema, encontramos x = 6 e y = 3. Logo, o lado do quadrado IJKL, que a hipotenusa do tringulo retngulo, mede

6 2 + 32 = 45 = 3 5 cm.

OUTRA SOLUO: O quadrado IJKL e o quadrado MNOP tm como lados as

hipotenusas dos tringulos retngulos dados, logo tm a mesma rea s. Fazendo os dois quadrados coincidirem, conclumos que o dobro da soma t das reas dos quatro tringulos retngulos a diferena entre as reas dos quadrados IJKL e EFGH, ou seja, 2t = 92 32 , o que fornece t = 36.. Assim, s = 9 + 36 = 81 36 = 45 cm2 e o lado do quadrado IJKL

45 = 3 5 cm.

03. Olhando para o ltimo nmero da fila n, vemos que ele a soma de todos os nmeros de 1 a n: por exemplo, na fila 4, o ltimo nmero da fila 1 + 2 + 3 + 4 = 10. Note que para obter a quantidade de nmeros at uma certa fila, basta somar o nmero da fila ao total de nmeros que havia antes dessa fila. Assim, temos, fila 5 : 15, fila 6: 21, fila 7: 28, fila 8: 36, fila 9: 45, fila 10: 55, fila 11: 66, fila 12: 78, fila 13: 91, fila 14: 105 O nmero de fitas adesivas horizontais entre uma fila n 1 e uma fila n igual a n 1 e o nmero de fitas adesivas verticais numa fila n igual n 1. Portanto, at a fila nmero 14, o nmero de fitas

(1 + 2 +  + 13) + (1 + 2 +  + 13) = 2

13 14 = 182. 2

04. Primeira Soluo: Unindo os pontos mdios de lados consecutivos do quadriltero, obtemos segmentos paralelos s suas diagonais e iguais metade delas. Portanto, o quadriltero assim obtido um paralelogramo. Os
EUREKA! N24, 2006

24

Sociedade Brasileira de Matemtica

segmentos traados dividem cada um dos quatro lotes em duas partes. Todas as partes internas tm a mesma rea s, igual a 1/4 da rea do paralelogramo. Cada uma das partes externas tem rea igual a 1/4 do tringulo determinado pela diagonal correspondente. Assim, a + c igual metade da rea do quadriltero, o mesmo ocorrendo com b + c. Da, a + s + c + s = b + s + d + s. Portanto, a rea S desconhecida satisfaz S + 210 = 200 + 250, ou seja, S = 240.

b a s s d s c s

Segunda Soluo: Ligando o ponto de interseo das retas que representam as duas cercas aos vrtices, obtemos:
M A B

O Q

Observemos que, como AQ = QD e as alturas de OAQ e OQD que passam por O so iguais, as reas de OAQ e OQD so iguais. Analogamente, as reas de OAM e OMB; OBN e ONC; OCP e OPD so iguais. Logo rea OAQ + rea OAM + rea OCP + rea ONC = rea OQD + rea OMB + rea OPD + rea OBN rea AMOQ + rea CNOP = rea DPOQ + rea BMON rea AMOQ = 200 + 250 210 = 240. 05. Como a + 3 mltiplo de 11, a + 3 = 11b, b Z. Sendo a mltiplo de 5, tambm , de modo que 3 = a 10b = b 3 b 5c b = 5c + 3 a = 11(5c + 3) 3 = 55c + 30, c ]+2 O nmero a + 2 mltiplo
EUREKA! N24, 2006

25

Sociedade Brasileira de Matemtica

de 9, assim como a + 2 54c 36 = c 4. Portanto c 4 = 9d c = 9d + 4 a = 55(9d + 4) + 30 = 495d + 250, d ]. Por fim, sendo a + 1 mltiplo de 7, ento a + 1 497d 245 = a + 1 7 (71d + 35) = 2d + 6 = 2(d 3) tambm , ou seja, d 3 = 7k d = 7 k + 3, k ] e a = 495(7 k + 3) + 250 = 3465t + 1735 Logo o menor valor de a 1735. Solues Nvel 2 Segunda Fase Parte B
SOLUO DO PROBLEMA 1:

Vamos representar por A, G e L a quantidade de questes de lgebra, Geometria e Lgica da Prova e por a, g e l as questes respondidas acertadamente em cada uma destas reas. As condies do problema fornecem as seguintes equaes: a g l a+l g+l = 0,5; = 0,7; = 0,8; = 0,62; = 0,74 A G L A+ L G+L Substituindo as relaes expressas pelas trs primeiras equaes nas outras duas, obtemos:

0,5 A + 0,8L 3L = 0,62 0,12 A = 0,18L A = A+L 2


0,7G + 0,8L 3L = 0,74 0,04G = 0,06 L G = G+L 2

A porcentagem de questes acertadas : a + g + l 0,5 A + 0,7G + 0,8 L = = A+G + L A+G + L


SOLUO DO PROBLEMA 2:

3 3 0,5. L + 0,7. L + 0,8L 2,6 2 2 = = 0,65 = 65% 3 3 4 L+ L+L 2 2

Vamos denotar por A, B, C e D os vrtices do quadrado e por MN o corte efetuado. Como CM + CN = BC = CD, resulta que BM = CN e DN = MC. Em conseqncia, os tringulos ADN e DCM so congruentes, o mesmo ocorrendo com ABM e BCN (em cada caso, os tringulos so retngulos e possuem catetos iguais). Logo, DN = CDM = e BM = CBN = . Assim, + + 27o = 90o e + = 63o.

EUREKA! N24, 2006

26

Sociedade Brasileira de Matemtica

A 27o

D N C

SOLUO DO PROBLEMA 3:

a) x2 9xy + 8y2 = x2 xy 8xy + 8y2 = x(x y) 8y (x y) = (x 8y)(x y). Alternativamente, as razes da equao do 2o grau x2 9xy + 8y2, de incgnita x, so y e 8y. Logo, x2 9xy + 8y2 fatora em (x 8y)(x y). b) A equao a ser resolvida (x y)(8y x) = 2005 (*) Observemos que a fatorao em primos de 2005 5 401. Alm disso, a soma dos fatores x y e 8y x 7y, que mltiplo de 7. A soma dos fatores 406, sendo que somente 406 mltiplo de 7. Assim, x y = 5 e 8 y x = 401 ou x y = 401 e 8 y x = 5 (*) ou x y = 5 e 8 y x = 401 ou x y = 401 e 8 y x = 5 x = 63 e y = 58 ou x = 459 e y = 58 ou x = 63 e y = 58 ou x = 459 e y = 58

As solues so, portanto, (63; 58), (459;58), (63; 58) e (459; 58).
OUTRA SOLUO:

Observando a equao dada como uma equao do segundo grau em x, obtemos x2 9yx + 8y2 + 2005 = 0 (*), cujo discriminante = (9y)2 4(8y2 + 2005) = 49y2 8020

EUREKA! N24, 2006

27

Sociedade Brasileira de Matemtica

Para que (*) admita solues inteiras, seu discriminante deve ser um quadrado perfeito; portanto 49y2 8020 = m2 (7y m)(7y + m) = 8020 = 22 5 401 (**) Podemos supor, sem perda de generalidade, que m 0, pois se (m; y) soluo de (**), ento ( m; y) tambm . Observando tambm que 7y m e 7y + m tm a mesma paridade e y m 7y + m, ento podemos dividir o problema em 4 casos: 7y m = 2 e 7y + m = 4010 m = 2004 e y = 2006/7, impossvel; 7y m = 10 e 7y + m = 802 m = 396 e y = 58; 7y m = 802 e 7y + m = 10 m = 396 e y = 58; 7y m = 4010 e 7y + m = 2 m = 2004 e y = 2006/7, impossvel.
9 y + m 9 58 + 396 = = 459 2 2

Se y = 58, as solues em x de (*) so


9 y m 9 58 396 = = 63 . 2 2

Se y = 58, as solues em x de (*) so e


9 y m 9 (58) 396 = = 459 . 2 2

9 y + m 9 (58) + 396 = = 63 2 2

Logo as solues so (63 ; 58), (459 ; 58), ( 63 ; 58) e ( 459 ; 58).


SOLUO DO PROBLEMA 4:

Veja a soluo do problema No. 3 do Nvel 1 parte B Solues Nvel 3 Segunda Fase Parte A Problema Resposta 01 12 02 1735 03 240 04 2011 05 2

01 . Primeiro observamos que os ngulos internos de um pentgono regular (5 2) 180 medem = 108 . 5
EUREKA! N24, 2006

28

Sociedade Brasileira de Matemtica

Como AF = AE = AB, o tringulo ABF issceles com

l ) = m( AFE l ) m( AFB l ) = 60 6 = 54 e No tringulo PEF, m( EFP l ) = 180 m( PEF l ) m( E FP l ) = 180 60 12 54 = 54 , ou seja, o m( EPF
l

l l l l ) = m( AFB l ) = 180 m( BAF ) = 180 m(B AE ) m(E AF ) = 180 108 60 = 6 . m( ABF 2 2 2

tringulo PEF issceles com PE = EF. Assim, como EF = AE, o tringulo PEA

l ) = 180 m( PEA) = 180 12 = 84. AE ) = m( E PA tambm issceles com m( P l 2 2 l ) 180 108 180 m( ABC Alm disso, m(C l AB) = = = 36 e 2 2 l ) = 108 36 = 72. m(C l AE ) = m( B l AE ) m(C AB AC ) = m( P l AE ) m(C l AE ) = 84 72 = 12. Logo, m( P l
02. PRIMEIRA SOLUO: Como a + 3 mltiplo de 11, a + 3 = 11b, b ] . Sendo a mltiplo de 5, a 10b = b 3 tambm , de modo que b 3 = 5c b = 5c + 3 a = 11(5c + 3) 3 = 55c + 30, c ] O nmero a + 2 mltiplo de 9, assim como a + 2 54c 36 = c 4. Portanto c 4 = 9d c = 9d + 4 a = 55(9d + 4) + 30 = 495d + 250, d ]. Por fim, sendo a + 1 mltiplo de 7, ento a + 1 497d 245 = a + 1 7 (71d + 35) = 2d + 6 = 2(d 3) tambm , ou seja, d 3 = 7k d = 7 k + 3, k ] e a = 495(7 k + 3) + 250 = 3465t + 1735. Logo o menor valor de a 1735.
SEGUNDA SOLUO:

As condies do problema equivalem a dizer que 2a 5 = 2(a + 1) 7 = 2(a + 2) 9 = 2(a + 3) 11 mltiplo de 5, 7, 9 e 11, donde mltiplo de 5 7 9 11 = 3465. Assim, o menor valor de a tal que 2a 5 = 3465 , ou seja, a = 1735. 03. Ligando o ponto de interseo das retas que representam as duas cercas aos vrtices, obtemos:

EUREKA! N24, 2006

29

Sociedade Brasileira de Matemtica

M A

O Q

Observemos que, como AQ = QD e as alturas de OAQ e OQD que passam por O so iguais, as reas de OAQ e OQD so iguais. Analogamente, as reas de OAM e OMB; OBN e ONC; OCP e OPD so iguais. Logo rea OAQ + rea OAM + rea OCP + rea ONC = rea OQD + rea OMB + rea OPD + rea OBN rea AMOQ + rea CNOP = rea DPOQ + rea BMON rea AMOQ = 200 + 250 210 = 240. 04. Substituindo y por 2 e x por a f(2) = a 8, obtemos f(a f(2) + f(2)) = a 8 + f ( f (2)) f(a) = a 8 + f(8). Substituindo a por 2 na ltima equao, obtemos f(2) = 2 8 + f(8) 8 = 2 8 + f(8) f(8) = 14. Assim f(a) = a 8 + 14 = a + 6 e f(2005) = 2005 + 6 = 2011. 05. A idia da soluo perguntar o valor numrico de p(k) para k suficientemente grande. Suponha que o polinmio seja: p(x) = an xn + an1 xn 1 + ... + a0, com an, an 1, ..., a0 inteiros positivos. Se k um inteiro, tal que: k > M = mx {an, an-1, ..., a0}, ento p(k) um inteiro, cujos dgitos na representao em base k so exatamente os coeficientes do polinmio p(x). Podemos ento tomar k igual a uma potncia de 10 suficientemente grande. Logo para resolver o problema, basta perguntarmos o valor de p(1), assim obtemos uma cota superior para M, e ento perguntamos o valor de p(x) para x igual a uma potncia de 10 maior do que p(1). Portanto, o nmero mnimo de perguntas que devemos fazer, para garantir que o polinmio p(x) seja determinado sem sombra de dvidas, 2. Por exemplo: Se p(1) = 29, perguntamos p(100), digamos que p(100) = 100613. Ento o nosso polinmio p(x) = 10x2 + 6x + 13.

EUREKA! N24, 2006

30

Sociedade Brasileira de Matemtica

Solues Nvel 3 Segunda Fase Parte B


SOLUO DO PROBLEMA 1:

Temos 9 xy x 2 8 y 2 = 2005 xy x 2 + 8 xy 8 y 2 = 2005 x( y x) + 8 y ( x y ) = 2005 ( x y )(8 y x) = 2005(*) Observemos que a fatorao em primos de 2005 5 401. Alm disso, a soma dos fatores x y e 8y x 7y, que mltiplo de 7. Devemos ento escrever 2005 como produto de dois fatores, cuja soma um mltiplo de 7. Para isso, os fatores devem ser 5 e 401. A soma dos fatores 406. x y = 5 e 8 y x = 401 ou x y = 401 e 8 y x = 5 (*) ou x y = 5 e 8 y x = 401 ou x y = 401 e 8 y x = 5 x = 63 e y = 58 ou x = 459 e y = 58 ou x = 63 e y = 58 ou x = 459 e y = 58

As solues so, portanto, (63; 58), (459;58), (63; 58) e (459; 58).
OUTRA SOLUO:

Observando a equao dada como uma equao do segundo grau em x, obtemos x2 9yx + 8y2 + 2005 = 0 (*), cujo discriminante = (9y)2 4(8y2 + 2005) = 49y2 8020 Para que (*) admita solues inteiras, seu discriminante deve ser um quadrado perfeito; portanto 49y2 8020 = m2 (7y m)(7y + m) = 8020 = 22 5 401 (**) Podemos supor, sem perda de generalidade, que m 0, pois se (m; y) soluo de (**), ento ( m; y) tambm . Observando tambm que 7y m e 7y + m tm a mesma paridade e 7y m 7y + m, podemos dividir o problema em 4 casos: 7y m = 2 e 7y + m = 4010 m = 2004 e y = 2006/7, impossvel;

EUREKA! N24, 2006

31

Sociedade Brasileira de Matemtica

7y m = 10 e 7y + m = 802 m = 396 e y = 58; 7y m = 802 e 7y + m = 10 m = 396 e y = 58; 7y m = 4010 e 7y + m = 2 m = 2004 e y = 2006/7, impossvel.
9 y + m 9 58 + 396 = = 459 2 2

Se y = 58, as solues em x de (*) so


9 y m 9 58 396 = = 63 . 2 2

Se y = 58, as solues em x de (*) so e


9 y m 9 (58) 396 = = 459 . 2 2

9 y + m 9 (58) + 396 = = 63 2 2

Logo as solues so (63 ; 58), (459 ; 58), ( 63 ; 58) e ( 459 ; 58).


SOLUO DO PROBLEMA 2:

c b 2 )2 b
) ) )

a2 ) 2

Podemos supor, sem perda de generalidade, a configurao acima e, portanto, pelo teorema de Pitgoras: 4(b2a2 b2A2 a2A2 + A4 ) = A4 + a4 + b4 + c4 2a2A2 2b2A2 + 2c2A2 + 2a2b2 2a2c2 2b2c2 3A 4 2 ( a 2 + b2 + c 2 ) A 2 ( a 4 + b 4 + c 4 2 a 2b 2 2 a 2 c 2 2b 2 c 2 ) = 0 O discriminante da equao do segundo grau acima, em
2 2 2 2 4 4 4 2 2

A2 + (

b2 A 2 a 2 A 2

) =c
2

(b A )(a A ) = a
2 2 2 2

+ b2 c 2 A 2

2 2 = 2( a + b + c ) + 4 3 ( a + b + c 2a b 2 a c 2b c ) = 2 2

EUREKA! N24, 2006

32

Sociedade Brasileira de Matemtica

16(a 4 + b4 + c 4 a 2 b 2 a 2 c 2 b 2 c 2 ). Logo

A 2 = 2(a
2

+ b2 + c 2 ) 16( a 4 + b4 + c 4 a 2 b2 a 2 c 2 b2 c 2 ) 2 3

+ b 2 + c 2 ) 2 a 4 + b 4 + c 4 a 2b 2 a 2 c 2 b 2 c 2 3 De fato, observando que A menor ou igual a min {a, b, c}, temos 2 2 2 A 2 a + b + c . Portanto 3

A 2 = (a

A=

( a 2 + b 2 + c 2 ) 2 a 4 + b 4 + c 4 a 2b 2 a 2c 2 b 2c 2 . 3

Observao: Outra maneira de obter as equaes trabalhar em R3, supondo, sem A A 3 , , com perda de generalidade, que C = (0, 0, 0), A = (A,0, h) e B = , 2 2 z h, z 0 . Obteramos, ento, as equaes A 2 + h 2 = a 2 , A 2 + z 2 = b 2 e A 2 + ( z h )2 = c 2 , que nos leva mesma equao da soluo acima. Curiosidade: Para o tringulo 3, 4, 5 a medida do lado da projeo que um tringulo equiltero aproximadamente e. O erro de apenas 0,1%.
SOLUO DO PROBLEMA 3:

Primeira Soluo: Seja an o nmero de ordenadas de resultados (sem derrotas), cujo total de pontos seja n. A pergunta do problema : quanto vale a20? Para responder a tal pergunta, iremos determinar uma relao recursiva entre os termos dessa seqncia. Pensando no ltimo resultado de uma ordenada de resultados totalizando n pontos, ele pode ser E ou V. Se for E, ento retirando o ltimo termo da ordenada, ela passa a totalizar n 1 pontos. Se for V, ento ao retiramos o ltimo resultado, a ordenada passa a totalizar n 3 pontos. Disto, conclumos que: an = an 1 + an 3. Calculando os valores da seqncia, temos: a1 = 1, a2 = 1, a3 = 2, a4 = 3, a5 = 4, a6 = 6, a7 = 9, a8 = 13, a9 = 19, a10 = 28, a11 = 41, a12 = 60, a13 = 88, a14 = 129, a15 = 189, a16 = 277, a17 = 406, a18 = 595, a19 = 872 e a20 = 1278.
EUREKA! N24, 2006

33

Sociedade Brasileira de Matemtica

Logo existem 1278 possveis seqncias ordenadas de resultados que o Flameiras pode ter obtido. Segunda Soluo: Sejam x e y o nmero de vitrias e empates do Flameiras, respectivamente. Temos que: x 0, y 0 x + y = 20. Dividindo em 7 possveis casos: 1 caso: x = 0 e y = 20: Temos exatamente uma seqncia ordenada de resultados. 2 caso: x = 1 e y = 17: Uma seqncia ordenada dever conter exatamente um V e 17 E, portanto o nmero de seqncias ordenadas exatamente o nmero de anagramas da palavra: VEEEEEEEEEEEEEEEEE, que : (17 + 1)! / (17! 1!) = 18. 3 caso: x = 2 e y = 14: Analogamente ao 2 caso, o nmero de seqncias ordenadas igual ao nmero de anagramas da palavra VVEEEEEEEEEEEEEE, que : (14 + 2)! / (14! 2!) = 120. 4 caso: x = 3 e y = 11: (11 + 3)! / (11! 3!) = 364 seqncias ordenadas. 5 caso: x = 4 e y = 8: (8 + 4)! / (8! 4!) = 495 seqncias ordenadas. 6 caso: x = 5 e y = 5: (5 + 5)! / (5! 5!) = 252 seqncias ordenadas. 7 caso: x = 6 e y = 2: (2 + 6)! / (2! 6!) = 28 seqncias ordenadas. Temos um total de 1 + 18 + 120 + 364 + 495 + 252 + 28 = 1278 seqncias ordenadas de resultados possveis.
SOLUO DO PROBLEMA 4:

Seja p, p + d, p + 2d, p + 3d, p + 4d, p + 5d, p + 6d a progresso aritmtica, que podemos supor crescente sem perda de generalidade. Ento: 1) p 2. De fato, se p = 2, p + 2d par e maior do que 2 e, portanto, no primo. 2) d mltiplo de 2. Caso contrrio, como p mpar, p + d seria par e maior do que 2. 3) p 3 Seno, teramos p + 3d mltiplo de 3, maior do que 3. 4) d mltiplo de 3 Caso contrrio, p + d ou p + 2d seria mltiplo de 3 e maior do que 3. 5) p 5 Seno teramos p + 5d mltiplo de 5, maior do que 5.
EUREKA! N24, 2006

34

Sociedade Brasileira de Matemtica

6) d mltiplo de 5. Caso contrrio, p + d , p + 2d, p + 3d ou p + 4d seria mltiplo de 5, maior do que 5. De 1), 2), 3), 4), 5) e 6), p 7 e d mltiplo de 30. Se p = 7, observando que 187 = 11 17, ento d 120. Para d = 120, a seqncia 7, 127, 247, 367, 487, 607, 727 a qual no serve, pois 247 = 13 19. Para d = 150, a seqncia 7, 157, 307, 457, 607, 757, 907 e satisfaz as condies do problema. Finalmente, se p 7, ento d mltiplo de 210 e o menor ltimo termo possvel para tais seqncias 11 + 6 210 = 1271. Portanto a resposta 907.

EUREKA! N24, 2006

35

Sociedade Brasileira de Matemtica

XXVII OLIMPADA BRASILEIRA DE MATEMTICA


Problemas e Solues da Terceira Fase
PROBLEMAS NVEL 1
PROBLEMA 1

Esmeraldinho tem alguns cubinhos de madeira de 2 cm de aresta. Ele quer construir um grande cubo de aresta 10 cm, mas como no tem cubinhos suficientes, ele cola os cubinhos de 2 cm de aresta de modo a formar apenas as faces do cubo, que fica oco. Qual o nmero de cubinhos de que ele precisar? Num tabuleiro quadrado 5 5 , sero colocados trs botes idnticos, cada um no centro de uma casa, determinando um tringulo. De quantas maneiras podemos colocar os botes formando um tringulo retngulo com catetos paralelos s bordas do tabuleiro? Observao: Tringulo retngulo todo tringulo que possui um ngulo de 90o. Os lados que formam esse ngulo so chamados de catetos.
PROBLEMA 3 PROBLEMA 2

A partir da casa localizada na linha 1 e na coluna 50 de um tabuleiro 100 100 , sero escritos os nmeros 1, 2, 3, 4, ..., n, como na figura a seguir, que apresenta uma parte do tabuleiro e mostra como os nmeros devero ser colocados. O nmero n ocupar a casa da linha 1, coluna 100.
Linha 100

  
46 45 44 43 42 41 40 39 38 37 47 29 28 27 26 25 24 23 22 48 30 16 15 14 13 12 11

  

Linha 10

Linha 1

... Coluna 1

49 31 17 7 6 5 4

50 32 51 18 33 8 19 2 9 1 3 Coluna 50

52 34 20 10

53 35 21

54 36

55

n Coluna 100 ...

EUREKA! N24, 2006

36

Sociedade Brasileira de Matemtica

a) Determine n. b) Em qual linha e coluna aparecer o nmero 2005?


PROBLEMA 4

No retngulo ABCD, com diagonais AC e BD, os lados AB e BC medem, respectivamente, 13 cm e 14 cm. Sendo M a interseco das diagonais, considere o tringulo BME, tal que ME = MB e BE = BA, sendo E A . a) Calcule a rea do tringulo BME. b) Mostre que o segmento BD paralelo ao segmento EC.
PROBLEMA 5

Um nmero inteiro positivo n tem a propriedade P se a soma de seus divisores positivos igual a 2n . Por exemplo: 6 tem a propriedade P, pois 1 + 2 + 3 + 6 = 2 6 , porm 10 no tem a propriedade P, pois 1 + 2 + 5 + 10 2 10 . Mostre que nenhum quadrado perfeito tem a propriedade P. Observao: Um nmero inteiro positivo um quadrado perfeito se igual ao quadrado de um inteiro. Por exemplo, 1 = 12 , 4 = 2 2 e 9 = 3 2 so quadrados perfeitos.

PROBLEMAS NVEL 2 Num tabuleiro quadrado 5 5 , sero colocados trs botes idnticos, cada um no centro de uma casa, determinando um tringulo. De quantas maneiras podemos colocar os botes formando um tringulo retngulo com catetos paralelos s bordas do tabuleiro?
PROBLEMA 2 PROBLEMA 1

No tringulo retngulo ABC, os catetos AB e BC medem, respectivamente, 3 cm e 4 cm. Seja M o ponto mdio da hipotenusa AC e seja D um ponto, distinto de A, tal que BM = MD e AB = BD. a) Prove que BM perpendicular a AD. b) Calcule a rea do quadriltero ABDC.

EUREKA! N24, 2006

37

Sociedade Brasileira de Matemtica

PROBLEMA 3

Dado que

(a b)(b c )(c a) 1 a b c + + ? = , qual o valor de a+b b+c c+a (a + b)(b + c)(c + a ) 11

PROBLEMA 4

Em seu treino dirio de natao, Esmeraldinho percorre vrias vezes, com um ritmo constante de braadas, o trajeto entre dois pontos A e B situados na mesma margem de um rio. O nado de A para B a favor da corrente e o nado em sentido contrrio contra a corrente. Um tronco arrastado pela corrente passa por A no exato instante em que Esmeraldinho sai de A. Esmeraldinho chega a B e imediatamente regressa a A. No trajeto de regresso, cruza com o tronco 6 minutos depois de sair de A. A seguir, Esmeraldinho chega a A e imediatamente sai em direo a B, alcanando o tronco 5 minutos depois da primeira vez que cruzou com ele ao ir de B para A. Quantos minutos o tronco leva para ir de A at B?
PROBLEMA 5

Prove que o nmero 12005 + 22005 + 32005 + ... + 20052005 mltiplo de 1 + 2 + 3 + ... + 2005.
PROBLEMA 6

A medida do ngulo B de um tringulo ABC 120. Sejam M um ponto sobre o lado AC e K um ponto sobre o prolongamento do lado AB, tais que BM a bissetriz interna do ngulo ABC e CK a bissetriz externa correspondente ao ngulo ACB. O segmento MK intersecta BC no ponto P. Prove que APM = 30. PROBLEMAS NVEL 3
PROBLEMA 1:

Um nmero natural palndromo quando se obtm o mesmo nmero ao escrevermos os seus dgitos na ordem inversa. Por exemplo, 481184, 131 e 2 so palndromos. Determine todos os pares de inteiros positivos (m, n) tais que 111...1 
111...1 

m uns n uns

palndromo.
PROBLEMA 2:

Determine o menor nmero real C para o qual a desigualdade

EUREKA! N24, 2006

38

Sociedade Brasileira de Matemtica


2005 2005 2005 2005 125 + x3 + x4 + x5 x1 x 2 x3 x 4 x5 x1 + x 125 + x 125 + x 125 + x 125 C x12005 + x 2 2 3 4 5 vlida para todos os nmeros reais positivos x1, x2, x3, x4, x5.

16

PROBLEMA 3:

Dizemos que um quadrado est contido em um cubo quando todos os seus pontos esto nas faces ou no interior do cubo. Determine o maior A > 0 tal que existe um quadrado de lado A contido num cubo de aresta 1.
PROBLEMA 4:

Temos quatro baterias carregadas, quatro baterias descarregadas e um rdio que necessita de duas baterias carregadas para funcionar. Supondo que no sabemos quais baterias esto carregadas e quais esto descarregadas, determine o menor nmero de tentativas suficiente para garantirmos que o rdio funcione. Uma tentativa consiste em colocar duas das baterias no rdio e verificar se ele, ento, funciona.
PROBLEMA 5:

Sejam ABC um tringulo acutngulo e F o seu ponto de Fermat, isto , o ponto B , BF C e CF A medem 120 interior ao tringulo ABC tal que os trs ngulos AF graus. Para cada um dos tringulos ABF, ACF e BCF trace a sua reta de Euler, ou seja, a reta que liga o seu circuncentro e o seu baricentro. Prove que essas trs retas concorrem em um ponto.
PROBLEMA 6:

Dados a, c inteiros positivos e b inteiro, prove que existe x inteiro positivo tal que a x + x b (mod c ) , ou seja, existe x inteiro positivo tal que c um divisor de ax + x b. SOLUES NVEL 1
PROBLEMA 1: SOLUO DE DANIEL LUCAS FILGUEIRA (FORTALEZA - CE)

Como cada cubinho tem 2 cm de aresta e o cubo tem 10 cm de aresta, ento cabem 5 cubinhos no comprimento, na largura e na altura, ento em todo o cubo cabem 125 cubinhos. Se no lado do cubo coubessem n cubinhos, ento o No. de cubinhos da parte de dentro do cubo seria (n 2) (n 2) (n 2). Como no lado do cubo cabem 5 cubinhos, ento para sabermos o No. de cubinhos da parte de dentro, basta substituir o n pelo 5, e ficaria o seguinte: (5 2) (5 2) (5 2) = 3 3 3 = 27
EUREKA! N24, 2006

39

Sociedade Brasileira de Matemtica

Como em todo o cubo cabem 125 cubinhos, ento para deixar o cubo oco, basta tirar a parte de dentro, que tem 27 cubinhos. Logo, Esmeraldinho precisaria de 125 27 = 98 cubinhos para formar o cubo oco.
PROBLEMA 2: SOLUO DE RAFAEL SUSSUMU YAMAGUTI MIADA (CAMPINAS - SP)

Se o boto correspondente ao ngulo reto estiver em (1, 1) teremos mais 4 casas acima e 4 casas direita, portanto 4 4 = 16 possibilidades. Se ele estiver em (2, 1) teremos mais 4 casas acima, 3 casas direita e 1 casa esquerda o que d de novo 4 4 = 16 possibilidades. Do mesmo modo, vemos que, para cada casa escolhida para o boto correspondente ao ngulo reto temos 16 possibilidades, e como no campo existem 25 casas, teremos portanto 25 16 = 400 possibilidades. Ento teremos 400 possibilidades.
PROBLEMA 3: SOLUO DA BANCA

a) Quando for escrito o nmero n, todas as casas da diagonal que passa pela (linha 100; coluna 1) e (linha 1; coluna 100) e as que esto abaixo dela estaro preenchidas e, nesse caso, 100 + 99 + 98 +...+ 3 + 2 + 1 = 5050 nmeros tero sido escritos no tabuleiro. Como comeamos com o 1, o ltimo, n, ser 5050. b) A quantidade de termos nas camadas (1, 2, 3), (4, 5, 6, 7, 8, 9, 10), (11, 12, 13, 14 , 15, 16, 17, 18, 19, 20, 21) aumenta de 4 em 4. Ao final da 31a camada, que tem 3 + 30 4 = 123 nmeros, tero sido escritos 3 + 7 + 11 +...+ 123 = 1953 nmeros, ou seja, o ltimo nmero dessa camada 1953. O termo que ocupa a linha mais alta em cada camada aumenta de 2 em 2 (veja que a 1a camada sobe at alinha 2, a 2a camada at a linha 4, a 3a sobe at a linha 6, e assim por diante). Assim, o termo da 31a camada que ocupa a linha mais alta estar na linha 1 + (122 2) = 62. Por fim, a 32a camada iniciar na linha 1 e coluna 51 32 = 19, com o nmero 1954, e subir at a linha 62 + 2 = 64. Como 2005 = 1954 + 51, o nmero 2005 aparecer na linha 51 + 1 = 52 e coluna 19.
PROBLEMA 4: SOLUO MATHEUS BARROS DE PAULA (TAUBAT - SP)

a) Montando a figura, ela ficar assim:


E 13 B 13 M

A 14 D

EUREKA! N24, 2006

40

Sociedade Brasileira de Matemtica

Os tringulos BEM e BAM so congruentes pelo critrio LLL. Como a distncia de M ao lado AB metade do lado AD, o tringulo BAM possui uma base de 13 cm e 13 7 uma altura de 7cm, e sua rea de = 45,5 cm2. 2 b) O tringulo BEM congruente ao tringulo CMD pelo critrio LLL, logo a HJJG HJJG distncia de E reta BD idntica distncia de C reta BD , pois as alturas sero as mesmas. Assim, EC // BD.
PROBLEMA 5: BASEADA NA SOLUO DE GUSTAVO LISBA EMPINOTTI (FLORIANPOLIS - SC)

HJJG HJJG

Um quadrado perfeito sempre tem um nmero mpar de divisores, pois h pares de nmeros cujo produto o quadrado perfeito dado e mais um nmero, a sua raiz. Se o quadrado perfeito n for mpar, ento todos os seus divisores so mpares, e assim ser sua soma. Logo a soma no pode ser 2n, pois 2n par. Se o quadrado perfeito n for par, ento igual a uma potncia de 2 vezes o quadrado de um mpar. Os divisores mpares de n so divisores desse quadrado e, como j vimos, sua soma (de todos os divisores mpares de n) mpar e logo a soma de todos os divisores de n tambm mpar, no podendo ser igual a 2n, que par. Portanto nenhum quadrado perfeito tem a propriedade P.

SOLUES NVEL 2
PROBLEMA 1: SOLUO DE HENRIQUE POND DE OLIVEIRA PINTO (SALVADOR BA)

Ao invs de considerarmos um tabuleiro quadrado consideremos uma malha pontilhada onde os pontos so centros de cada quadradinho. Isto :

EUREKA! N24, 2006

41

Sociedade Brasileira de Matemtica

Observe que para cada tringulo do enunciado existe um nico conjunto dos 2 pontos extremos da hipotenusa. Ou seja, o conjunto de dois pontos extremos da hipotenusa no tringulo abaixo {8; 20}.
1 6 11 16 21 2 7 12 17 22 3 8 4 9 5 10 15 20 25

13 18 23

14 19 24

Ou seja os pontos 8 e 20 determinam a hipotenusa do tringulo abaixo. No entanto para cada dois pontos que determinam a hipotenusa existem outros dois pontos que podem ser o vrtice oposto hipotenusa. No exemplo acima os pontos 8 e 20 determinam a hipotenusa de dois tringulos retngulos; os tringulos 8; 18; 20 e 8; 10; 20. Basicamente, cada tringulo possui uma nica hipotenusa e cada hipotenusa comum a dois tringulos retngulos do enunciado. Para provar que cada hipotenusa pertence a dois tringulos retngulos distintos, vamos pegar uma hipotenusa genrica de extremos (5K + a) e (5 N + b) (no quadriculado acima) contando que ambos sejam menores que 25 e tanto a e b sejam maiores ou iguais a 1 e menores ou iguais a 5. Observe que K N e a b ou seja, ambos os pontos esto em linhas e colunas diferentes pois se coincidirem em linhas e ou colunas no h tringulos como definidos no enunciado com essa hipotenusa. Os dois e (5K + a; 5K + b; 5 N + b) tringulos com essa hipotenusa so: (5K + a; 5 N + a; 5N + b ) como para cada tringulo h uma nica hipotenusa e para cada hipotenusa dois tringulos, o nmero de tringulos o dobro do de hipotenusas. Vamos calcular o nmero de hipotenusas: O primeiro ponto pode ficar em 25 lugares (todos os pontos) j o segundo pode ficar em 16 (todos que no esto na mesma linha ou coluna do primeiro). Logo so 25 16 = 400 onde a ordem das escolhas importa, mas a ordem no importa. Logo, como so duas escolhas dividi-se por 2! = 2 e teremos 400/2 = 200 hipotenusas 400 tringulos. Logo a resposta 400 tringulos. Obs. Para um quadrado n n a quantidade de tringulos n 2 ( n 1) 2 2 = n (n 1) se generalizamos esse processo que foi utilizado. 2
2

EUREKA! N24, 2006

42

Sociedade Brasileira de Matemtica

PROBLEMA 2: SOLUO DE RAFAEL TUPYNAMB DUTRA (BELO HORIZONTE - MG)


B 90 2 D

A M

l = . Chamemos BCA AB 3 Temos sen = = AC 5 BC 4 cos = = . AC 5


a) Como ABC retngulo em B, sabemos que B pertence circunferncia de dimetro AC. Desta forma, AM = MB = MD AM = MD. Isso significa que M pertence mediatriz de AD . Como AB = BD, B tambm pertence mediatriz de AD . HJJJ G Assim, a mediatriz de AD a reta BM e, assim, BM AD. c) D pertence circunferncia de centro M e raio BM , j que BM = MD. Assim, D pertence ao circuncrculo do ABC. Como AB = BD, temos AB = BD e, l = BCA l BCD l = . No tringulo ABC , B l assim, BCD AC = 90 . Logo, como

F F

l = 180 (90 ) = 90 + . No tringulo BCD, temos ABDC inscritvel, BDC l = 180 (90 + ) = 90 2. C BD
rea BCD = BD BC sen (90 2 ) 2 =

3cm 4cm cos 2 = 6cm 2 cos 2 e, como 2

3 4 sen = e cos = , 5 5

EUREKA! N24, 2006

43

Sociedade Brasileira de Matemtica

4 2 3 2 16 9 42 2 rea BCD = 6cm (cos sen ) = 6cm = 6cm2 = cm 5 5 25 25 Usamos aqui o fato de que cos 2 = cos 2 sen2 . AB BC 3cm 4cm = = 6cm 2 rea ABC = 2 2 42 192 2 rea ABDC = rea ABC + rea BCD = 6 + cm2 = cm . 25 25
2 2 2 2

PROBLEMA 3: SOLUO ADAPTADA DE MARCELO MATHEUS GAUY (SO JOS DO RIO PRETO - SP)

Inicialmente, podemos observar que a+b b+c c+a b c a b c a + + =3 + + = 3 + + , a+b b+c c+a a+b b+c c+a a +b b +c c +a a b c + + ou seja, obter o valor de = equivalente a obter o valor de a+b b+c c+a b c a = + + . a+b b+c c +a Como j sabemos que = 3 , basta agora conseguir outra relao entre e aproveitando a igualdade fornecida no enunciado a qual envolve =

( a b ) (b c ) ( c a ) ( a + b ) (b + c ) ( c + a )

Aps alguns testes, substituindo valores em a, b e c, somos levados a supor que ab bc ca ab bc ca + + = = , isto , . a+b b+c c+a a +b b +c c +a (Temos acima uma incrvel identidade, ela fornece infinitas triplas de reais cuja soma igual ao oposto do produto!). Vamos demonstrar tal identidade: a b b c c a ( a b)(b + c)(c + a) + (b c)( a + b)(c + a) + (c a)( a + b)(b + c) + + = (*) a+b b+ c c+ a ( a + b)(b + c)(c + a) Porm, 1) (a b)(b + c)(c + a ) = (a b )(b c + 2c)( a c + 2c ) =

(a b)(b c )(a c ) + 2c ( a b)(b c + a c + 2c) = (a b)(b c )(a c ) + 2c ( a b)(a + b)

EUREKA! N24, 2006

44

Sociedade Brasileira de Matemtica

2)

(b c)( a + b)(c + a) + (c a)( a + b)(b + c) = (a + b)((b c)(c + a ) + (c a )(b + c)) = (a + b)(bc + ba c2 ca + cb + c2 ab ac) = (a + b)( 2bc 2ac) = 2c (a b)( a + b)

Logo, de 1 e 2, ( a b)(b c)(c a) . (*) = ( a + b)(b + c)(c + a) Assim, = 1 1 16 (3 ) = = . 11 11 11

PROBLEMA 4: SOLUO DE HENRIQUE WATANABE (SO PAULO - SP)

Vamos supor que a velocidade da corrente do rio c e a velocidade de Esmeraldinho v (sem a corrente). Seja d o comprimento do rio. Em 6 minutos os dois juntos percorreram 2d. A velocidade no sentido A B (v + c ) e a velocidade no sentido B A

( v c) .

O primeiro encontro foi 6c do ponto A. d De A B ele leva minutos. v+c O segundo encontro ocorreu 11c do ponto A. 2dv d d + = Para ir e voltar Esmeraldinho leva: t = ( v + c v c v + c)(v c) e de A at 11c: 2dv + 11c (v c) 11c . Logo = 11 v+c (v + c)(v c)

2dv + 11cv 11c2 = 11v2 11c2 v (11v 11c 2d ) = 0 De B at o primeiro encontro em 6c t =

( d 6c)(v + c) + d (v c) d 6c . Logo 6 = v c (v + c)(v c )

6v2 6c2 = dv + dc 6vc 6c2 + dv dc 2v (3v + 3c d ) = 0 Como v 0: 11v 11c 2d = 0 3v + 3c d = 0 11v 11c = 2d 6v + 6c = 2d

EUREKA! N24, 2006

45

Sociedade Brasileira de Matemtica

17c 5 17c 51c + 15c 66c Ento: d = 3 + 3c = = 5 5 5 d 66c 1 66 minutos O tronco leva: t = = = 5 c 5 c 66 minutos, ou seja, 13 minutos e 12 segundos. O tronco vai de A para B em 5 11v 11c = 6v + 6c v =
PROBLEMA 5: SOLUO DE HENRIQUE POND DE OLIVEIRA PINTO (SALVADOR - BA)

Observe que 1 + 2 + 3 + ... + 2005 =

2005 ( 2005 + 1) 2

= 2005 1003

Seja E = 12005 + 22005 + ... + 20052005 Vamos provar que 2005| E Vendo E mdulo 2005 temos: E 12005 + 22006 +32005 +... +10012005 +10022005 + (1002)2005 + (1001)2005 +... +(2)2005 +(1)2005 + 02005
2005

como a2005 ( a )
2005

2005

a2005 + (a) 2005 0 Temos que o n-simo termo da


2005

expresso acima ir se anular com o (2005 n) e, portanto, E 0(mod2005) 2005| E . Vamos provar agora que 1003 divide E. Vendo E mdulo 1003 temos: E 12005 + 22005 + ... +10012005 +10022005 + 02005 + (1002)2005 + (1001)2005 + ... + (2)2005 + (1)2005
1003

como a2005 (a)2005 a2005 + (a)2005 0 cada n-simo termo ir se anular com o
1003 1003

(2006 n) termo e o 1003o. j mltiplo de 1003 pois igual a 10032005 temos que E 0(mod1003) 1003 | E com o 1003 | E e 2005 | E e (1003,2005 ) = 1 1003 2005 | E 1 + 2 + ... + 2005 |12005 + 22005 + ... + 20052005 c.q.d.

EUREKA! N24, 2006

46

Sociedade Brasileira de Matemtica

PROBLEMA 6: SOLUO DE MARLIA VALESKA COSTA MEDEIROS (FORTALEZA - CE)


A

60 B

Q 60 30 30

M 30 P

30

I L

Observe a figura acima: Vamos explicar como chegar at ela! Sejam: Q o ponto de interseco de BM e AP .

l , que encontra o I a interseco de CK e a bissetriz externa de ABC prolongamento de AC em L.

l KCL l = (pelo enunciado) BCK


Vamos provar que A, Q, P, I so colineares. Usando Menelaus no ABC, temos: M, P e K so colineares

AK BP CM = 1 (*) BK CP AM
S temos que: Pelo Teorema da bissetriz interna: (I) No ABC CM BC = AM AB Pelo Teorema da bissetriz externa:
EUREKA! N24, 2006

47

Sociedade Brasileira de Matemtica

(II) No ABC AK AC = BK BC Ento, substituindo em (*):

Com este resultado, observe que AP bissetriz do ngulo B l AC . Como Q a interseco de AP e BM , Q pertence a bissetriz do ngulo B l AC , o l . que implica, que no ABM , AQ bissetriz de B AM l . Vamos provar que I pertence a bissetriz de BAM Pelo teorema da bissetriz interna: (III) No BCK CI BC = IK BK Por (II), temos

AC BP BC AC BP AC CP =1 =1 = BC CP AB AB CP AB BP

AK BK IK IK CI = = = AC BC CI AK AC
Observe que isto nada mais nada menos do que a propriedade da bissetriz l = BAM l . interna. Logo AI bissetriz do ngulo K AC Assim, provamos que A, Q, P e I so colineares. l = . No ABC podemos observar que 2 + = 60 Seja ACB S que:

l = 120 + 2 (teorema BCL


l = 2 = 60 + BCL

do

ngulo

externo).

Sabemos,

ento,

que

 K = 120 Logo, AI Olhando para o quadriltero BPIK, observe que este inscritvel, pois: l + PIK  = 60 + 120 = 180 PBK
l I PK l = 30 Assim, I BK l I PK l (o.p.v) Agora, observe que APM l Portanto, APM = 30

 K = + + = + + 60 + = 2 + + 60 Observe que o ngulo AI

EUREKA! N24, 2006

48

Sociedade Brasileira de Matemtica

SOLUES NVEL 3
PROBLEMA 1: SOLUO DE WILSON CAMARA MARRIEL (RIO DE JANEIRO - RJ)

Podemos escrever 11...1 como


k

Ento:

N
n

10k 1 . 9

m m n n 1 n n n 1 10m (10n 1) 10n 1 10 1 10 1 11...100...0 11...1 11...1099...988...89 9 9 11...1 11...1 = = = = 9 9 9 9 9 m n m

NN

P  P P  

se supusermos m > n (o que no nos faz perder a generalidade!). Esse nmero tem m + n 1 algarismos. Ento vejamos: (fazendo n > 9)

11...1099...988...89 k

2  
n 1 m n n 1

9 1234567901
n 2

! x 

onde x o ltimo termo do quociente antes de usar o 0 e k o resto de 11...1 por 9.


n 1 m n n 1

Isto , acima temos parte da diviso euclidiana de 11...1099...988...89 por 9. Dividimos nos casos para possveis valores de k. 1o. caso (k = 0) n = 9 + 1. Continuando a diviso, obtemos

2  

2
n1

0099...988...89

   
mn n 1

9 1234567901
n 2

x 011111 ! !10987 


mn

Contando da esquerda para a direita 8 o (m + 1)o. algarismo; como m + n 1 m +1> ento est na metade direita do resultado. Logo, j que o 8 no 2 aparece no lado esquerdo e aparece no direito, temos uma assimetria, ABSURDO! 2o. caso (k = 1): n = 9 +2
EUREKA! N24, 2006

49

Sociedade Brasileira de Matemtica

1099...988...89 19 19 18 088 mesmo caso anterior, ABSURDO! 3o. caso (k = 2): n = 9 +3


mn

   
m n n 1

9 1234567901
n 2

! 22098 ! x 122 


mn

2099...988...89 29 29 28 18 088 mesmo caso anterior, ABSURDO!

   
n 1

9 1234567901
n 2

! ! 32098 x 233 


mn

Ao tentarmos os 9 casos, vemos que um deles tem que ser 9. Basta conferir se verdade! Vejamos: n = 1 verdade! 11...11 n = 2 111...1 verdade! 122...21 11...11 111...1 verdade! Obs. fcil ver que sempre dar certo de 1 9. n=3 1111...1 1233...321

EUREKA! N24, 2006

50

Sociedade Brasileira de Matemtica

11...111111111 11111111...111 n = 9 verdade! Pois basta ir somando 1 aos 111111111...11 1111111111...1 1234567899...987654321 posteriores. Ento nossos pares so:

(1, k ), ( 2, k ), (3, k ), ( 4, k ), (5, k ), (6, k ), (7, k ), (8, k ) , (9, k ) ( m, n) = tal que k < *. (k ,1), (k ,2), (k ,3), (k ,4), (k ,5), (k ,6), (k ,7), (k ,8), (k ,9)
PROBLEMA 2: SOLUO DE ANDERSON HOSHIKO AIZIRO (SO PAULO SP)

Para x1 = x2 = x3 = x4 = x5 = 1 temos C 5 516 C 515. Por bunching (ou desigualdade de Muirhead), temos que

x
sym

1 2 3 4 5 1 2 3 4 5

1 2 3 4 5 x x x x x1 x2 x3 x4 x5 sym

no caso de termos uma desigualdade homognea e simtrica (o que o nosso caso). Alm disso, devemos ter 1 1 , 1 + 2 1 + 2 , 1 + 2 + 3 1 + 2 + 3 , 1 + 2 + 3 + 4 1 + 2 + 3 + 4 e 1 + 2 + 3 + 4 + 5 = 1 + 2 + 3 + 4 +5 . A notao

x
sym

1 2 3 4 5 1 2 3 4 5

x x x x

significa que estamos somando todos os 5! = 120

1 2 3 4 5 xi2 xi3 xi4 xi5 , sendo (i1 , i2 , i3 , i4 , i5 ) uma permutao de (1, 2, 3, termos da forma xi 1

4, 5). Aqui, impomos tambm 1 2 3 4 5 e o mesmo para os ' s. Observando x1 x2 x3 x4 x5 x

que +x
125 2

a +x
125 3

desigualdade +x
125 4

125 1

+x

125 16 5

dada

simtrica,

se

abrirmos

obtemos um somatrio simtrico no qual o

maior expoente de algum dos termos 125 16 + 1 < 2005. Podemos, ento, aplicar bunching. A desigualdade , ento, equivalente a C
EUREKA! N24, 2006

1 516 2005 x1 x1 1 x 2 2 x 3 3 x 4 4 x5 5 4! sym 5! sym

51

Sociedade Brasileira de Matemtica

Explicando:

x
sym

2005 1

possui 5! termos no desenvolvimento no qual temos 4!

2005 2005 2005 2005 , 4! x3 , 4! x4 e 4!x5 . x12005 , 4!x2

Alm

(x

disso,

temos

125 1

125 125 + x125 2 + x3 + x4 + x

125 16 5

516

termos

no

desenvolvimento

de

e, para cada conjunto (1 , 2 , 3 , 4 , 5 ) temos 5! 516 somatrios simtricos. 5!


2 3 4 5 x11 x2 x3 x4 x5 , se C sym

dos 516 termos e, portanto, h

Como por bunching desigualdade vlida.

x
sym

2005 1

1 516 = C = 515 4! 5!

Desse modo, conclumos que o menor nmero real C para o qual a desigualdade
2005 2005 2005 2005 125 125 125 125 + x3 + x4 + x5 x1 x2 x3 x4 x5 x1 + x125 C x12005 + x2 2 + x3 + x4 + x5

16

vlida

para todos os nmeros reais positivos x1 , x2 , x3 , x4 , x5 C = 515.

PROBLEMA 3: SOLUO DA BANCA

Primeiro, mostremos que podemos supor, sem perda de generalidade, que os centros do cubo (que doravante chamaremos C) e do quadrado coincidem. Suponha que os centros no coincidam. Considere os trs planos distintos, cada um deles paralelo a duas faces do cubo, que passam pelo centro do quadrado. Os trs planos determinam no cubo oito paraleleppedos; considere o de menores dimenses (ou seja, algum que tem todas as dimenses menores ou iguais a 1/2). Seja a a maior dimenso desse paraleleppedo. Ento construa um cubo C0 de lado 2a com centro no centro do quadrado e faces paralelas s faces do cubo do problema. O quadrado est contido nesse cubo, pois cada plano ou contm o quadrado ou o corta em dois polgonos congruentes e simtricos em relao ao centro do quadrado. Translade o cubo C0, incluindo o quadrado, que est em seu interior, de modo que o centro de C0 coincida com o centro do cubo. Agora os centros do quadrado e de C coincidem, e dado que 2a 1, C0 est contido em C, e o quadrado ainda est contido no cubo C. A figura a seguir mostra que
EUREKA! N24, 2006

A 3 42 .
52

Sociedade Brasileira de Matemtica

Note

que
2

3 2 3 1 1 , AD = BC = 2 AB = CD = + 12 + = 4 4 4 4

3 3 2 1 2. AC = BD = + 12 + 12 = = 2 2 4
1 4 3 4

3 4

D 1
4

1 B 4 3 4

3 4

1 4

Vamos provar que, na verdade,

A > 3 42 . Podemos supor que o centro do quadrado coincide com o centro do


cubo. Seja S uma esfera com centro no centro O de C e que passa pelos quatro 2 / 2 > 3/ 4. A figura a seguir mostra as vrtices do quadrado, ou seja, de raio seces de S no cubo C. Numeramos as oito regies contidas na superfcie da esfera e no interior do cubo com nmeros romanos

A = 3 42 . Suponha que exista um quadrado de lado A

EUREKA! N24, 2006

53

Sociedade Brasileira de Matemtica

IV

III

VIII

VI V VII II

Agora, vamos tentar localizar os vrtices do quadrado de lado

A > 3 42

em S.

Note que cada um dos quatro vrtices deve pertencer a uma das regies de I a VIII. Suponhamos, sem perda de generalidade, que dois vrtices opostos do quadrado esto contidos nas regies I e, conseqentemente, II, j que vrtices opostos do quadrado so diametralmente opostos em S. Considere o paraleleppedo de menores dimenses que contm as regies I e, digamos, III. Sejam x, x e 1 as suas dimenses. Vamos provar que dois pontos no 3 2 . interior desse paraleleppedo esto a uma distncia menor que 4
x x I 1

III

EUREKA! N24, 2006

54

Sociedade Brasileira de Matemtica

Primeiro, considere uma face do cubo e sua interseo com a esfera. A partir da figura a seguir, podemos concluir que o raio da esfera 1 1 1 y2 + + = y2 + . 2 2 2
2 2 2

Como

raio

da

esfera

maior

que

3 2 2 3 2 1 3 1 1 1 = , y + > y > . Conseqentemente, x = y < . 4 2 4 2 4 4 2 4


y y

1 2

1 3 2 e, portanto, +1 = 16 4 dois vrtices do quadrado no podem estar contidos em I e III. Como um dos vrtices pertence a I, no pode existir vrtice do quadrado em III e, analogamente, em IV e V. Da mesma forma, lembrando que um dos vrtices do quadrado est em II, no pode haver vrtices do quadrado em VI, VII e VIII. Mas ento no sobraram regies para os outros dois vrtices do quadrado, absurdo. 3 2 . Deste modo, o maior lado de um quadrado contido no cubo unitrio = 4 A diagonal do paraleleppedo mede x 2 + x 2 + 12 < 2

PROBLEMA 4: SOLUO DE GABRIEL TAVARES BUJOKAS (SO PAULO SP)

Separe as pilhas em 3 grupos, 2 de 3 pilhas e 1 de 2 pilhas, e faa todas as 3 3 2 possibilidades dentro dos grupos. Faremos + + = 7 tentativas. 2 2 2 Como so 3 grupos e 4 baterias boas, um grupo ter 2 baterias boas, e em algum momento sero testadas juntas.
EUREKA! N24, 2006

55

Sociedade Brasileira de Matemtica

Suponha que seja possvel com apenas 6 tentativas. Considere o grafo com 8 vrtices representando as 8 baterias e as arestas {i, j} representando que a bateria i 8 e a j no foram testadas juntas. O grafo tem 6 = 22 arestas. 2 Por Turn, ele tem um K4. De fato, o mximo de arestas sem K4 8 3 3 2 = 21 . 2 2 2 2 Se as 4 baterias carregadas forem as respectivas aos vrtices do K4, temos que nunca duas delas foram testadas juntas, logo o algoritmo com 6 tentativas falha nesta situao. Resposta: Com 7 tentativas possvel. Observao: o grafo completo de n vrtices (tambm conhecido como n-clique), notado por Kn, um grafo em que todo par de vrtices ligado. O teorema de Tran diz que, fixado o nmero de vrtices n, o grafo que no contm um Kr com a maior quantidade possvel de arestas o grafo (r 1)-partido completo com classes de vrtices as mais distribudas possveis. Esse grafo obtido da seguinte forma: divida os vrtices em r 1 conjuntos, de modo que a diferena entre as quantidades de vrtices nos conjuntos seja no mximo 1; em seguida, ligue todo par de vrtices que no esto no mesmo conjunto. Na aplicao do problema 4, Gabriel dividiu os 8 vrtices em trs grupos com 3, 3 8 3 3 2 e 2 vrtices. Da a quantidade de arestas mxima ser 2 = 21 . 2 2 2

Alis, a resoluo de Gabriel tambm resolve uma generalizao do problema, no qual h m baterias funcionando e n baterias descarregadas. Tente pensar nesse problema!

EUREKA! N24, 2006

56

Sociedade Brasileira de Matemtica

PROBLEMA 5: SOLUO DE LEANDRO FARIAS MAIA (FORTALEZA CE)

Construa um tringulo equiltero BXC, externo a ABC. O ponto O1 o circuncentro do BFC e tambm de BXC . G o baricentro do ABC . AG XO1 =2= O1G // XF. Mas: O3 A = O3 F e O2 A = O2 F Temos: GM O1M AF O2 O3 O1G O2 O3 . Analogamente temos: O2G O1O3 e O3G O1O2 G o ortocentro do O1O2O3 .
A O3 O2

G1 B C

Sendo G1 o baricentro do FBC temos:

FG1 AG =2= G1G // AF G1M GM G1G O2O3 como G o ortocentro de O1O2O3 , ento G1 est na altura relativa a O2O3 . Portanto, O1G1 , O2 G 2 e O3G3 so concorrentes em G (seu ortocentro).

Lema: Seja p primo e a, b, r, inteiros, a > 0, > 0. Ento existe x > 0 tal que x r (mod p 1) a x + x b (mod p ) Demonstrao: Induo em . Para = 1 (Base): x r (mod p 1) , que tem soluo pelo teorema chins Se p | a , ento obtemos x b (mod p ) dos restos. Se p / | a; x = ( p 1)(ar b + r + l c) + r; com l tal que x > 0. De fato, x r (mod p 1) e
EUREKA! N24, 2006

PROBLEMA 6: SOLUO DE GABRIEL TAVARES BUJOKAS (SO PAULO SP)

57

Sociedade Brasileira de Matemtica

a x + x ar ( a p1 )
p

(a b+r +l c)
r

+ ( p 1)( ar b + r + l c ) + r ar ar + b r l c + r b.
p p

Passo: Da hiptese, existe x0 tal que a

x0

+ x0 = b + t p

e x0 r

(mod p 1) .

Tomando x1 = x0 + ( p 1) p t : x1 x 0 r e a x1 + x1 a x0 a ( p 1) p +1
p

) +x
t

p 1

p 1

+ ( p 1) p t a x0 + x 0 p t = b +1
p

Isso termina a demonstrao do lema.


2 n ... pn ; p1 < p2 < ... < pn a fatorao em primos de c. Seja c = p11 p2

Vamos mostrar por induo em n que x tal que a x + x b (mod c n ) , onde


2 ... pii . ci = p11 p2

Base: n = 1: a x + x b mod p11 . Caso especial do lema. Passo: Se xi tal que a xi + xi b ( mod ci ) , pelo lema existe x tal que x x i (mod pi +1 1) a x + x b (mod p i +i1+1 ) Pelo teorema chins dos restos xi +1 tal que xi +1 x i (mod mmc( pi +1 1; ci ; (c i )) ) xi +1 x (mod p i +i1+1 )

i +1 Observe que mdc p i +1 , mmc( p i +1 1; ci ; (c i )) = 1 pois pi + 1 maior que todos os fatores primos de ci e, conseqentemente, de (c i ) . i +1 Logo xi +1 x i x (mod p i +1 1) e xi +1 x (mod p i +1 )

xi +1 x (mod ( p i +1 1) pi +i1+1 ) , ou seja, xi +1 = x + p i +i1+1 ( pi +1 1)l '


x+ p Assim, a xi+1 + xi +1 a i+1 + pi+i1 1
i +1

( pi+1 1)l '

+x ax + x be + +
pi+i1 1 pi+i1 1 ci

x + c l a xi+1 + xi +1 a i ( i ) + xi + ci k a xi + xi b onde (ci ) l = k ci = xi +1 xi . ci ci

b a xi+1 + xi +1 b. Portanto a xi+1 + xi +1 +


pi+i1 1 ci ci+1

Em especial, a xn + xn b a xn + xn b .
cn c

EUREKA! N24, 2006

58

Sociedade Brasileira de Matemtica

XXVII OLIMPADA BRASILEIRA DE MATEMTICA Primeira Fase Nvel Universitrio

PROBLEMA 1

Seja f : definida por f ( x) = x 3 + ax 2 + bx + c , sendo a, b e c inteiros. Sabe-se que f(1) = f(1) = 0. As retas tangentes ao grfico de f nos pontos A = (1; 0) e B = (1; 0) cortam-se em C. Calcule a rea do tringulo ABC, sabendo-se que tal rea inteira.
PROBLEMA 2
4

\ \

Calcule a integral:
PROBLEMA 3

ln(1 + tgx)dx

Determine o maior valor possvel para o volume de um tetraedro inscrito no x2 y2 z 2 elipside de equao + + =1. 9 16 25
PROBLEMA 4

Sejam A e B matrizes reais quadradas de mesma dimenso tais que, para todo inteiro positivo k, ( A + B ) k = A k + B k . Prove que se A invertvel ento B a matriz nula.
PROBLEMA 5

Determine todos os valores reais de para os quais a matriz A = (a ij ) nn definida por aij = cos((i 1) j), para 1 i, j n, tem determinante nulo.
PROBLEMA 6

Prove que existem pelo menos 2005 potncias 27-simas distintas (isto , nmeros da forma n27, com n inteiro positivo), todas com exatamente 2005 algarismos, tais que qualquer uma pode ser obtida de qualquer outra a partir de uma permutao de seus algarismos.

EUREKA! N24, 2006

59

Sociedade Brasileira de Matemtica

XXVII Olimpada Brasileira de Matemtica GABARITO Primeira Fase


Solues Nvel Universitrio
SOLUO DO PROBLEMA 1:

Pelo enunciado, temos f(x) = (x 1)(x + 1)(x c) = x3 cx2 x + c, f'(x) = 3x2 2cx 1, donde f '( 1) = 2(1 + c) e f '(1) = 2(1 c). Assim, as equaes das retas AC e BC so, respectivamente, y = 2(1 + c)(x + 1) e y = 2(1 c)(x 1). Igualando para obter as coordenadas de C, temos (1 + c)(x + 1) = (1 c)(x 1) x = 1/c y = 2(c + 1)(c 1)/c Assim a rea pedida S = |2(c + 1)(c 1)/c|, pois o tringulo ABC tem base AB = 2 e altura y = 2(c + 1)( c 1) / c . Como c e a rea S so inteiros, temos c | 2(c + 1)(c 1). Mas (c + 1) e (c 1) so primos com c, donde c | 2. Assim c = 1 ou c = 2. Os casos c = 1 do S = 0, um tringulo degenerado. Os casos c = 2 do S = 3. O valor da rea , portanto, igual a 3.
SOLUO DO PROBLEMA 2:

senx senx + cos x Temos ln(1 + tgx ) = ln 1 + = ln . cos x cos x 2 Entretanto, sen x + = senx cos + sen cos x = (senx + cos x), e logo 4 4 4 2 senx + cos x = 2 sen x + . 4 2 sen x + 4 ln 2 = Assim, ln(1 + tgx ) = ln + ln sen x + ln cos x, donde cos x 2 4
EUREKA! N24, 2006

60

Sociedade Brasileira de Matemtica

ln (1 + tgx ) dx =

ln 2 4 + ln sen x + dx 4 ln cos x dx. 0 0 8 4

Agora, sen x + = cos x + = cos x , donde 4 2 4 4


4 4 + = = ln sen ln cos ln cos y dy x dx x dx 0 0 0 4 4 ln 2 (fazendo a substituio y = x ), donde 4 ln (1 + tgx ) dx = . 0 4 8 4

SOLUO ALTERNATIVA DO PROBLEMA 2:

x, du = dx. 4 0 1 tgu 2 Ento I = ln 1 + tg u ( du ) = 4 ln 1 + du = 4 ln du 0 0 4 1 + tgu 1 + tgu 4 Seja I = 4 ln(1 + tgx )dx; faa u =
0

= 4 ln 2du 4 ln(1 + tgu ) du =


0 0

ln 2 ln 2 ln 2 I 2I = I = . 4 4 8

SOLUO DO PROBLEMA 3:

LEMA: O tetraedro de maior volume inscrito na esfera unitria x2 + y2 + z2 = 1 o tetraedro regular. Seus vrtices podem ser tomados como ( c, c, c) com um nmero par de sinais onde c = 3 3 . Sua aresta a = 2 6 3 e seu volume V = 8 3 27. O elipside do problema obtido a partir da esfera unitria aplicando a transformao linear 3 0 0 T = diag (3,4,5) = 0 4 0 . Tetraedros inscritos na esfera so levados em 0 0 5 tetraedros inscritos no elipside multiplicando o volume por |det (T)| = 60. Assim um tetraedro de volume mximo (3c, 4c, 5c), com um nmero par de sinais , de volume160 3 9. Demonstrao do LEMA: A nica parte no trivial a de provar que um tetraedro de volume mximo deve ser regular. Vamos provar que todas as faces de um tetraedro de volume mximo
EUREKA! N24, 2006

61

Sociedade Brasileira de Matemtica

so tringulos equilteros. Para isso vamos fixar o vrtice V0 e variar os vrtices V1, V2, V3 restritos ao crculo definido por estes pontos. Ora, com este tipo de mudana a altura do tetraedro no muda, donde maximizamos o volume maximizando a rea do tringulo V1, V2, V3. um fato sabido e de fcil demonstrao que o tringulo de rea mxima inscrito em um crculo dado o equiltero.
SOLUO DO PROBLEMA 4:

Temos, de A2 + B 2 = ( A + B) 2 = ( A + B)( A + B) = A2 + AB + BA + B 2 que AB + BA = 0. Agora, A3 + B3 = ( A + B)3 = ( A + B)( A + B) 2 = ( A + B)( A2 + B 2 ) = A3 + AB 2 + BA2 + B3 , donde AB 2 + BA2 = 0. Como BA = AB, 0 = AB 2 + BA2 = AB 2 ABA = A( B 2 BA) e, como A invertvel, B 2 BA = 0. Temos, tambm A3 + B 3 = ( A + B )3 = ( A + B ) 2 ( A + B) = ( A2 + B 2 )( A + B ) = A3 + A2 B + B 2 A + B 3 , donde A2 B + B 2 A = 0. Como B 2 = BA, segue que A2 B + BA2 = 0, e, como BA = AB, obtemos 0 = A2 B + BA2 = A2 B ABA = A( AB BA), donde AB = BA, pois A invertvel. Finalmente, de AB + BA = 0, segue que 2 AB = 0, donde, como A invertvel, devemos ter B = 0.
SOLUO DO PROBLEMA 5:

Sabemos que para todo k Pk (t ) = ck ,k t + ... + ck ,1t + ck ,0

natural

existe

um

polinmio

de grau k tal que cos(ka ) = Pk (cos a ) para todo a.


2 Por exemplo, P0 = 1, P 1 = t, P 2 = 2t 1 Temos portanto aij = Pi 1 (cos( j)) = ci 1, k (cos( j ))k 0 k <i

Podemos agora, para i > 1, subtrair ci 1,0 vezes a primeira linha da i-sima linha sem alterar o determinante obtendo assim que, para i > 1,  ij = a ci 1, k (cos( j))k .
0< k < i

EUREKA! N24, 2006

62

Sociedade Brasileira de Matemtica

Para i > 2, subtramos ci 1,1 vezes a segunda linha da i-sima linha, ainda sem alterar o determinante. Repetindo o processo, vemos que det(A) = det(B) onde bij = ci 1,i 1 (cos( j ))i 1 Assim, a menos dos fatores ci 1,i 1 , B uma matriz de Vandermonde, e seu determinante igual a ( j1 + j0 ) ci 1,i 1 (cos( j1 ) (cos( j0 )) =(2)n(n1)/ 2 ci 1,i 1 sen 2 1< j in j0 < j1 i n ( j1 j0 ) sen . 2 Assim det(A) = 0 se e somente se existem 1 j0 < j1 n tais que ( j + j ) ( j j ) sen 1 0 = 0 ou sen 1 0 . 2 2 Mas isto ocorre se e somente se ( j1 j0 ) = 2k , k inteiro. Ou seja, det(A) = 0 se e somente se = 2k /( j1 j0 ) para alguma escolha de 1 j0 < j1 n Falta verificar quais os valores possveis de j1 j0 . Para n 1 o problema trivial (det(A) = 1), donde no h nenhum com essa propriedade. Para n = 2, os nicos valores possveis de j1 j0 so 1 e 3, 2k , com k inteiro donde deve ser da forma 3 Para n > 2, j1 j0 assume todos os valores inteiros positivos m at 2n 1 , donde deve ser da forma 2k m , com m 2n 1 e k inteiro. Observao: Temos ainda ck ,k = 2k 1 para k > 1 donde

c
i n

i 1,i 1

= 2( n 1)( n 2) / 2 e
2

det( A) = (1) n ( n 1) / 2 2( n 1) .

sen
j1 < j0

( j1 + j0 ) ( j1 j0 ) sen 2 2

EUREKA! N24, 2006

63

Sociedade Brasileira de Matemtica

Demonstrao da afirmao cos(ka) = Pk(cos(a)) [No vale pontos extras]: Temos cos((k + 1)a ) + cos((k 1)a ) = 2cos(ka ) cos a, donde, assumindo que o resultado vale para k 1 e para k, cos((k + 1)a ) = 2cos a Pk (cos a) Pk 1 (cos a ), o que prova o resultado fazendo Pk +1 ( x) = 2 xPk ( x) Pk 1 ( x), para k 1, com P0 ( x) = 1 e P 1 ( x ) = x. Note que, sabendo que o coeficiente lder ck , k de P k ( x) 2k 1 , segue imediatamente que o coeficiente lder ck +1, k +1 de Pk +1 ( x) 2 2k 1 = 2k = 2( k +1)1. 06. Vamos estimar inicialmente a quantidade de tipos de nmeros de 2005 algarismos a menos de uma permutao de seus algarismos. Um tal tipo de nmeros est determinado pelas quantidades x0 , x1 ,..., x9 de algarismos iguais a 0, 1, , 9, respectivamente; devemos ter x0 + x1 + ... + x9 = 2005. Assim, a quantidade desses tipos de nmeros , no mximo, o nmero de solues de x0 + x1 + ... + x9 = 2005, com xi 0 para 0 i 9, que 2005 + 9 2014 9 4 9 36 = < 2014 < (10 ) = 10 . 9 9 Por outro lado n 27 tem 2005 algarismos
2004 27 2005 27

se,

somente
2005 27

se,
2004 27

102004 n 27 < 10 2005 10 n < 10 , donde h pelo menos 10 27 naturais n tais que n tem 2005 algarismos. Entretanto,

10

2005 2004 2004 2004 2004 1 ln10 ln10 ln10 10 27 10 27 =10 27 1027 1 =10 27 e 27 1 >10 27 >1074 >1072 20051036 , 27 27 donde, pelo princpio da casa dos pombos, h pelo menos 2005 naturais n tais que n 27 tem 2005 algarismos e esses nmeros n 27 so todos do mesmo tipo (seus algarismos so os mesmos a menos de uma permutao).

Nota: possvel estimar 10 27 1 sem usar a desigualdade e x 1 x. Por exemplo:


1 1 1 1 1 16 10 > 10 = 10 2 > 316 > (1,7 )8 > (1,3) 4 > (1,12) 2 > 1,05, donde 1 1 (que foi o que usamos). 10 27 1 > 0,05 > 100 1 27 1 32 1

EUREKA! N24, 2006

64

Sociedade Brasileira de Matemtica

XXVII OLIMPADA BRASILEIRA DE MATEMTICA SEGUNDA FASE NVEL UNIVERSITRIO PRIMEIRO DIA
PROBLEMA 1:

Determine, em funo de n, o nmero de possveis valores para o determinante de A, dado que A uma matriz real n n tal que A3 A2 3 A + 2I = 0 , onde I representa a matriz identidade n n, e 0 representa a matriz nula n n.
PROBLEMA 2:

Sejam f e g funes contnuas distintas de [0, 1] em (0, + ) tais que n +1 1 f ( x) 1 1 , seja Para 0 = n y ( ) ( ) . = f x dx g x dx n 0 g ( x)n dx. 0 0 Prove que ( yn )n 0 uma seqncia crescente e divergente.
PROBLEMA 3:

Sejam v1 , v2 ,..., vn vetores em

tais que vi 1 para 1 i n e

v
i =1

= 0. Prove

que existe uma permutao de {1, 2,, n} tal que k com 1 k n. Obs. Se v = ( x, y )

v
j =1

( j )

5 para qualquer

\ ,v =
2

x 2 + y 2 denota a norma euclidiana de v.

SEGUNDO DIA
PROBLEMA 4:

Considere a seqncia (an )n 1 dada por a1 = 1, an +1 = an + Prove que a srie

1 a
2005 n

, n 1.

na
n =1

converge.

EUREKA! N24, 2006

65

Sociedade Brasileira de Matemtica

PROBLEMA 5:

Prove que

n
n =1

1
n

= x x dx.
1 0

PROBLEMA 6:

Prove que para quaisquer naturais 0 i1 < i2 < ... < ik e 0 j1 < j2 < ... < jk , a matriz i + js (ir + js )! A = (ars )1 r ,s k dada por ars = r (1 r , s k ) invertvel. = ir ! js ! ir
SOLUES PROBLEMA 1: SOLUO DE MOYSES AFONSO ASSAD COHEN (RIO DE JANEIRO RJ)

Podemos Podemos

escrever

A3 A2 3 A + 2I = 0

como

( A 2 I ) ( A2 + A I ) = 0.

Se provarmos que no existem duas combinaes ( d1 , d2 , d3 ) e ( e1 , e2 , e3 ) tais que


d2 d3 e2 e3 3 = 2e1 2 3 2d1 2 , ento o nmero de possveis valores para o determinante, ser o nmero de maneiras de escolher d1 , d 2 , d3 satisfazendo di > 0 e d1 + d2 + d3 = n. E o nmero de escolher esses di ' s

concluir ento que os possveis autovalores de A so 1 5 1 + 5 1 = 2, 2 = e 3 = . Seja di a multiplicidade do autovalor i . 2 2 d2 d3 3 , onde d1 + d2 + d3 = n. Temos que o determinante de A da forma: 2d1 2

n! (n + 2)(n + 1) n 2 + 3n + 2 = . 2 n!2! n! 2 d2 e1 e2 e3 3 d se, e somente se, d1 = e1 , Vamos provar ento que 2d1 2 3 = 2 2 3 d 2 = e2 e d3 = e3 .
d2 d3 e2 3 3 = 2e1 2 e 2d1 2 3 , d1 + d 2 + d 3 = e1 + e2 + e3 = n d2 n d1 d2 e2 n e1 e2 2d1 2 1 = 2e1 2 3 , Podemos usar tambm que 2 3 = 1,

( n + 2 )! =

1 + 5 1 5 +1 + 5 5 5 4 = = 1.) (pois = 2 2 4 4

EUREKA! N24, 2006

66

Sociedade Brasileira de Matemtica

1 2 n d1 d2 1 2 n e1 e2 e1 2 3 = 2 3 3 3
d1
d1 e1 ) + 2 ( d 2 e2 ) n d1 2 d 2 n e1 2 e2 2d1 ( 1) 2 3 = 2e1 ( 1) 2 3 2d1 e1 = ( 1) e2 d2 ( ,e 3 d e

como no podemos escrever 1 + 5 , onde I um inteiro maior que 1, como uma potncia de 2, temos que a igualdade verdadeira se, e somente se, os expoentes so zero. Ou seja d1 e1 = 0 d1 = e1 e portanto d3 = e3 . e2 d 2 = 0 d 2 = e2
PROBLEMA 2: SOLUO DE HUMBERTO SILVA NAVES (S.J. DOS CAMPOS SP)

Sejam: un = yn +1 yn =
0 1

f ( x ) n +1 [ f ( x) g ( x)] dx g ( x) n +1 f ( x) n +1 2 f ( x) g ( x ) ) dx n+ 2 ( ( ) g x

e vn = un+1 un =
0 1

Sabemos que vn > 0, n , pois

f ( x ) n +1 2 f ( x ) g ( x ) ) > 0, n+ 2 ( g ( x) x f g Para [0;1] e como e so contnuas, temos: f ( x ) n +1 2 f ( x ) g ( x ) ) dx > 0, n , logo (un )n 0 crescente. n +2 ( g ( x) 0 Vamos agora provar que u0 > 0 : vn =
1

u0 = u0 ( f ( x ) g ( x ) ) dx =
0 0

1 f ( x) f x g x dx ( ) ( ) ( ) [ f ( x) g ( x )]dx = g ( x) 0

=
0

( f ( x) g ( x))
g ( x)

dx > 0 un uo > 0, n , pois (un )u >0 crescente.

Portanto claramente yn yo + n uo , n 0, e ( yn )n 0 crescente e divergente.

EUREKA! N24, 2006

67

Sociedade Brasileira de Matemtica

PROBLEMA 3: SOLUO DA BANCA

Vamos usar a soluo da verso em R do problema: se 1 , 2 ,..., m so nmeros reais com i 1, i m e

i =1

= 0 ento existe

uma permutao de {1, 2,,m} tal que ( i ) tenha sinal contrrio a (i. e., com (i ) ( j ) 0).
j <i

j <i

( j )

Seja I = {1, 2,,n}. Escolhemos um conjunto X I tal que

v
iX

seja o maior

possvel. Podemos supor (rodando os eixos coordenados, se necessrio) que

um vetor da forma (0, y), com y > 0. Sejam 1 , 2 : 2 as projees na primeira e na segunda coordenadas, respectivamente. Usando a verso em do problema para reordenar os elementos de X e de I\X, podemos supor que X = {1,

\ \

v
iX

k +1 r 2,,k} para um certo k > n, 1 vi 1, r k e 1 v j 1, s n k . i =1 j = k +1 Note agora que 2 (vi ) 0, i X (e 2 ( vi ) 0, i I \ X ) , pois, se j X e 2 (v j ) < 0, teramos

verso em do problema) uma permutao de I que intercala os ndices em X e em I\X , preservando a ordem dos ndices em X e em I\X, de modo que m 2 v (i ) 1, m n. Como os ndices em X e em I\X aparecem em ordem, i =1 m teremos 1 v (i ) 1 v( i ) + 1 v (i ) 1 + 1 = 2, m n, e logo i i m i =1 i i m (i )I \ X

iX \{ j}

vi >

iX

, absurdo. Podemos ento obter (como na

i =1

(i )

22 + 12 = 5, m n.

EUREKA! N24, 2006

68

Sociedade Brasileira de Matemtica

PROBLEMA 4: SOLUO DE DIGO VELOSO UCHA (RIO DE JANEIRO RJ)

Primeiro recorde a expanso binomial de Newton: n n n ( a + b ) = a nk b k . k =0 k Agora usando isso verifique as seguintes contas: 1 an + (a + b ) = 2005 ( an ) (pois an sempre positivo) Logo: 2006 > a12006 + 2006 a2
2006 2006 2006 > a2 + 2006 a3 2006

= ( an )

2006

2006 2005 1 2006 + + ... > (an ) + 2006. an 2005 1 ( an )

( an +1 ) > ( an ) + 2006. 2006 2006 ( an+1 ) > ( a1 ) + ( 2006 ) n > (n + 1) 1/ 2006 1+ 1 an+1 > ( n + 1) ( n + 1) an+1 > ( n + 1) 2006
2006 2006
1 1 1 1 < < 1 1+ 1+ 1 2006 2006 ( n + 1) an+1 ( n + 1) n =1 n an n =1 ( n )

Como

1 com > 1 converge segue que n =1 n

na
n =1

tambm converge.
n

PROBLEMA 5: SOLUO DE LUS DANIEL BARBOSA COELHO (RIO DE JANEIRO - RJ)

=e

ln( x x )

=e

( x ) ln x

=
n=0

( x ) ( ln x )
n

n!

x dx =
1 x 0

n=0

( x ) ( ln x )
n

n!

dx

devido ao tipo de convergncia montona da srie de potncias e y =


n =0

yn , n!

podemos fazer troca da integral com o somatrio, obtendo:

x x dx =
n=0

( 1)
n!

x n ( ln x ) dx
n
1 n 0

( p) ( p) denotemos por I n a seguinte integral: I n = x p (ln x ) dx, para todo p inteiro

no negativo.
EUREKA! N24, 2006

69

Sociedade Brasileira de Matemtica

Integrando por partes: u = (ln x ) du =


n ( p) In =

n x p +1 n 1 ( ln x ) dx; dv = x p dx v = x p +1

1 (1) p +1 x p +1 n n n 1 n (ln x ) dx 1 x p (ln x ) dx = (ln1) 0 0 ( p + 1) p +1 p +1

lim +
x 0

x p +1 n n n n ( p) ( p) (ln x ) In In x p +1 (ln x ) = 0. 1 = 1 , pois lim + 0 x p +1 p +1 p +1

n ( p ) ( p) = In I n 1 p +1

1 ( p ) I1( p ) = I0 p +1 (1) n n! ( p ) (1)n n! 1 p (1) n n! ( p) = I0 = x dx = , tomando p = n temos: In n n 0 ( p + 1) ( p + 1) ( p + 1)n+1


1 0

x x dx =

(1)n n! n=0

(1)n n ! 1 1 . = = n +1 n +1 n n =1 n ( n + 1) n =0 ( n + 1)

PROBLEMA 6: SOLUO DE HUMBERTO SILVA NAVES (S.J. DOS CAMPOS SP) Considere os seguintes pontos no reticulado: An = ( in ;0) e Bn = ( 0; jn ) , onde

1 n k .
y

B4 B3 B2 B1 A4 A3 A2 A1 x

Um caminho ligando An com Bm um caminho no reticulado partindo de An e chegando em Bm que s pode ir para cima ou para a direita. Exemplos:

EUREKA! N24, 2006

70

Sociedade Brasileira de Matemtica


y
y

Bm 2
2

Bm

3 2 An

3 An

caminho!

No caminho!

i + j Um fato interessante que existem n m caminhos ligando An com Bm . in Uma rota uma coleo de k caminhos (de cada An parte exatamente um caminho e em cada Bm chega exatamente um caminho) e dizemos que uma rota bem feita se os caminhos no se cruzam em nenhum ponto do reticulado. Vamos provar que o nmero de rotas bem feitas igual a det(A). Pela definio de determinante, temos: k in + j ( n ) det( A) = (1) I ( ) . in S n =1
k

k i + j Mas (1) I ( ) n ( n ) exatamente o nmero de rotas ligando An com B( n) , in n =1 para 1 n k , multiplicando pela paridade I() da permutao . Vamos provar que as rotas mal feitas se cancelam neste somatrio:

y 4 X 3 2 Y 5 A4 4 A3 3 2 A2 A1
1

B4 B3 B2 B1 x

Considere uma rota mal feita R e seja Y o ponto de interseco com maior coordenada x (se existir mais de um, tome Y cuja coordenada y seja a menor possvel).

= {(4 ; 4) ; (3 ; 3) ; (2 ; 1) ; (2 ; 2)}

EUREKA! N24, 2006

71

Sociedade Brasileira de Matemtica

Vamos trocar os respectivos caminhos que se cruzam em Y (se existir mais de 2 caminhos que se cruzam em Y, troque os caminhos que comeam em An ' s de mais maior coordenada x). Assim obtemos uma nova rota mal y feita, s que com a paridade de permutao correspondente trocada 4 B4  X (1) I ( ) = (1) I ( ) 3 B3 Como a relao entre rotas mal 2 B2 feitas que acabamos de definir Y bijetora, ento provamos que as 1 B1 rotas mal feitas no contribuem para x 5 4 3 2 1 o somatrio, e como uma rota bem feita possui a identidade como A4 A3 A2 A1 permutao associada, provamos  = {(4 ; 4) ; (3 ; 3) ; (2 ; 1) ; (2 ; 2)} que: det (A) = nmero de rotas bem feitas. Como 0 i1 < i2 < ... < ik e 0 j1 < j2 < ... < jk , certamente o nmero de rotas bem feitas diferente de 0, pois uma rota bem feita.
y B4 B3 B2 Y B1 A1 x

A4

A3

A2

Errata: O item b) do problema No. 112 (Eureka! 23, p.60) foi proposto equivocadamente: ao contrrio do que pensvamos, parece no haver solues simples para ele. Gostaramos portanto de manter apenas o item a) do problema proposto No. 112. Pedimos desculpas pelos inconvenientes causados.

EUREKA! N24, 2006

72

Sociedade Brasileira de Matemtica

XXVII OLIMPADA BRASILEIRA DE MATEMTICA Resultado Nvel 1 (5a. e 6a. Sries)


NOME Matheus Barros de Paula Guilherme Vieira Melo Luis Musso Gualandi Rafael Dias da Fonsca Rodrigo Rolim Mendes de Alencar Gustavo Lisba Empinotti Iuri Rezende Souza Eduardo Cintra Simes Joo Mendes Vasconcelos Gabriel Lima Guimares Jonas Rocha Lima Amaro Bruno Cesar da Silva Guedes Kelve Torres Henrique Igor Rosiello Zenker Daniel Lucas Filgueira Cleiton Vilela Figueiredo da Silva Andreza Lais da Silva Nascimento Ivan Seki Hellmeister Matheus Henrique Botelho Cordeiro Breno Rocha Comin Henrique Lopes de Mello Leonardo Henrique Caldeira Pires Ferrari Leonardo Gonalves Fischer Francisco Vagner Dantas Leite Filho Elder Massahiro Yoshida Alex Lordello Magario Rafael Sussumu Yamaguti Miada Deborah Barbosa Alves Diogo Silva Freitas Matheus Barbosa Santos de Miranda Augusto dos Santos Morgan Andr Bandeira Pinheiro Fernando Fonseca Andrade Oliveira Eduardo F. Freire Neto Wellington Biing Jung Lee Mac'simus Alec'sander de Castro Duarte Alessandro Macdo de Arajo Camila Miraglia Ribeiro Douglas Barbosa da Fonsca Pedro Montebello Milani Gabriel Ricardo Loecsh Siebiger Tiago Yparraguirre Vigas Rafael de Melo Andrade Priscilla Lie Sato Yamaguti Joo Lucas Camelo S Franciely Juliani Chutti Frederico Nascimento Dutra Isaac Jernimo Moreira Anne Wang Rafael Farias Marinheiro Rafael Fernandes Paixo Filipe da Gama Martin Humberto Lopes Tabatinga Neto Gregory Cosac Daher Renata Aimi Fukuda Fabrcio Catani de Freitas Bruno Giordano Leite Victor Gonalves Elias Letcia Duchein Ferreira Larissa Firakawa Tamashiro Douglas Souza Alves Junior Lara Guimares Fernandes Peres CIDADE - ESTADO Taubat SP Fortaleza CE Vitria ES Arapiraca AL Fortaleza CE Florianpolis SC Mineiros GO Recife PE Fortaleza CE Vitria ES Fortaleza CE Recife PE Recife PE So Paulo SP Fortaleza CE Recife PE Recife PE So Paulo SP Curitiba PR Leme SP Rio de Janeiro RJ Rio de Janeiro RJ Fraiburgo SC Fortaleza CE So Paulo SP Salvador BA Campinas SP So Paulo SP Recife PE Joo Pessoa PB S. J. do Rio Pardo SP Fortaleza CE Belo Horizonte MG Salvador BA So Paulo SP Fortaleza CE Fortaleza CE Curitiba PR Arapiraca AL So Paulo SP Sobradinho DF Niteri RJ Boituba SP So Paulo SP Fortaleza CE Itajobi SP Porto Alegre RS Fortaleza CE So Paulo SP Recife PE Rio de Janeiro RJ Nanuque MG Teresina PI Rio de Janeiro RJ So Paulo SP Sorocaba SP Recife PE Joo Pessoa PB Londrina PR Jundia SP Vassouras RJ Rio de Janeiro RJ PRMIO Ouro Ouro Ouro Ouro Ouro Prata Prata Prata Prata Prata Prata Prata Prata Prata Prata Prata Prata Prata Bronze Bronze Bronze Bronze Bronze Bronze Bronze Bronze Bronze Bronze Bronze Bronze Bronze Bronze Bronze Meno Honrosa Meno Honrosa Meno Honrosa Meno Honrosa Meno Honrosa Meno Honrosa Meno Honrosa Meno Honrosa Meno Honrosa Meno Honrosa Meno Honrosa Meno Honrosa Meno Honrosa Meno Honrosa Meno Honrosa Meno Honrosa Meno Honrosa Meno Honrosa Meno Honrosa Meno Honrosa Meno Honrosa Meno Honrosa Meno Honrosa Meno Honrosa Meno Honrosa Meno Honrosa Meno Honrosa Meno Honrosa Meno Honrosa

EUREKA! N24, 2006

73

Sociedade Brasileira de Matemtica

Nvel 2 (7a. e 8a. Sries)


NOME Henrique Pond de Oliveira Pinto Marcelo Matheus Gary Rafael Tupynamb Dutra Pollyanna Stfani Borges Freitas Iuri Souza Ramos Barbosa Guilherme Philippe Figueiredo Marcelo Tadeu de Oliveira S Marlen Lincoln da Silva Henrique Watanabe Grazielly Muniz da Cunha James Jun Hong Pedro Pinheiro de Negreiros Bessa Marilia Valeska Costa Medeiros Camilla Matias Morais Mrcio Rabello de Freitas Alex Atsushi Takeda Renan Lima Novais Rafael Horimoto de Freitas Dielson de Britto Junior Hugo Fonseca Arajo Vitor Mori Cindy Yuchi Tsai Thiago Ribeiro Ramos Gabriel Moreira Francisco Tales Augusto Gonalves Alphonse Nathana Alcntara Lima Illan Feiman Halpern Thiago da Silva Pinheiro Jlio Czar Batista de Souza Thiago Ide Sousa Danilo Marcolongo Afonso Caio Jos Fonseca Santos Caio Srgio Parente Silva Isabella Amorim Gonalez Ana Lusa de Almeida Losnak Yuri Bastos Pereira Mateus Sampaio de Mendona Alisson de Brito Ninomia Alan Eduardo dos Santos Ges Jos Cabadas D. Neto Marcelo Rafael Silva Rempel Rafael Rabelo de Carvalho Davi Lopes Alves de Medeiros Gabriella Fonseca Ribeiro Christian Eduardo de Umeki e Saiki Marco Antonio Lopes Pedroso Catarina Yu Na Kim Rafael Alves da Silva Pedro Henrique Azevedo Damacena Renan Henrique Finder Ricardo Bioni Liberalquino Dalen Chen Kuang Izabela Karennina Travizani Maffra Jennifer Katherine Koshiba Yu Felipe Onrio da Silva Oliveira CIDADE ESTADO Salvador BA S. J. do Rio Preto SP Belo Horizonte MG Fortaleza CE Braslia DF Fortaleza CE Barreiras BA Fortaleza CE So Paulo SP Fortaleza CE So Paulo SP Fortaleza CE Fortaleza CE Fortaleza CE Mesquita RJ Londrina PR Niteri RJ So Paulo SP Rio de Janeiro RJ Juiz de Fora MG So Paulo SP So Paulo SP Varginha MG Santo Andr SP Paraguau Paulista SP Fortaleza CE Itatiaia RJ So Paulo SP Salvador BA Suzano SP S. B. do Campo SP Rio de Janeiro RJ Rio de Janeiro RJ Fortaleza CE So Paulo SP Rio de Janeiro RJ Belo Horizonte MG So Paulo SP Fortaleza CE Salvador BA Maring PR Braslia DF Fortaleza CE Betim MG So Paulo SP Santa Isabel SP So Paulo SP Teresina PI Fortaleza CE Joinville SC Macei AL Osasco SP Belo Horizonte MG So Paulo SP Botucatu SP PRMIO Ouro Ouro Ouro Ouro Ouro Ouro Prata Prata Prata Prata Prata Prata Prata Prata Prata Bronze Bronze Bronze Bronze Bronze Bronze Bronze Bronze Bronze Bronze Bronze Bronze Bronze Bronze Bronze Bronze Meno Honrosa Meno Honrosa Meno Honrosa Meno Honrosa Meno Honrosa Meno Honrosa Meno Honrosa Meno Honrosa Meno Honrosa Meno Honrosa Meno Honrosa Meno Honrosa Meno Honrosa Meno Honrosa Meno Honrosa Meno Honrosa Meno Honrosa Meno Honrosa Meno Honrosa Meno Honrosa Meno Honrosa Meno Honrosa Meno Honrosa Meno Honrosa

EUREKA! N24, 2006

74

Sociedade Brasileira de Matemtica

Nvel 3 (Ensino Mdio)


NOME Gabriel Tavares Bujokas Guilherme Rodrigues Nogueira de Souza Thoms Yoiti Sasaki Hoshina Regis Prado Barbosa Luty Rodrigues Ribeiro Rafael Mendes de Oliveira Cesar Ryudi Kawakami Jose Marcos Andrade Ferraro Jos Armando Barbosa Filho Anderson Hoshiko Aiziro Leandro Farias Maia Andr Linhares Rodrigues Levi Mximo Viana Leonardo Ribeiro de Castro Carvalho Wilson Camara Marriel Fabiano Edson Carlos Adenilson Arcanjo de Moura Junior Edson Augusto Bezerra Lopes Rodrigo Viana Soares Eduardo Fischer Rafael Sampaio de Rezende Rafael Montezuma Pinheiro Cabral Gustavo Sampaio Sousa Ramon Moreira Nunes Hector Kenzo Horiuti Kitahara Francisco Tarcsio Guedes Lima Verde Neto Alexandre Hideki Deguchi Martani Enzo Haruo Hiraoka Moriyama Rafael Morioka Oda Andr Lucas Ribeiro dos Santos Michel Faleiros Martins Antnio Felipe Cavalcante Carvalho Rafael Moura e Sucupira Artur de Almeida Losnak Tiago Porto Barbosa Willy George do Amaral Petrenko Douglas Bokliang Ang Cunha Breno Vieira de Aguiar Beatriz Laiate Vinicius Gripp Barros Ramos Lucio Eiji Assaoka Hossaka Mateus Oliveira de Figueiredo Marcus Edson Barreto Brito Flvio Henrique Moura Stakoviak Ricardo Turolla Bortolotti Pedro Henrique Silva Belisrio Filipe Alves Tom Frederico de Souza Frydman Heytor Bruno Nobre Pitombeira das Virgens Daniel Lopes Alves de Medeiros CIDADE ESTADO So Paulo SP So Paulo SP Rio de Janeiro RJ Fortaleza CE Fortaleza CE Rio de Janeiro RJ So Paulo SP So Paulo SP Fortaleza CE So Paulo SP Fortaleza CE Fortaleza CE Fortaleza CE So Paulo SP Rio de Janeiro RJ Fortaleza CE Fortaleza CE Fortaleza CE Fortaleza CE Encantado RS Fortaleza CE Fortaleza CE Fortaleza CE Fortaleza CE So Paulo SP Fortaleza CE So Paulo SP So Paulo SP So Paulo SP Pindamonhangaba SP Campinas SP Fortaleza CE Fortaleza CE So Paulo SP Fortaleza CE Rio de Janeiro RJ S. J. dos Campos SP Fortaleza CE Sorocaba SP Rio de Janeiro RJ Curitiba PR Fortaleza CE Fortaleza CE Belm PA Rio Claro SP Rio de Janeiro RJ Fortaleza CE Salvador BA Fortaleza CE Fortaleza CE PRMIO Ouro Ouro Ouro Ouro Ouro Ouro Prata Prata Prata Prata Prata Prata Prata Prata Prata Prata Bronze Bronze Bronze Bronze Bronze Bronze Bronze Bronze Bronze Bronze Bronze Bronze Bronze Bronze Bronze Bronze Meno Honrosa Meno Honrosa Meno Honrosa Meno Honrosa Meno Honrosa Meno Honrosa Meno Honrosa Meno Honrosa Meno Honrosa Meno Honrosa Meno Honrosa Meno Honrosa Meno Honrosa Meno Honrosa Meno Honrosa Meno Honrosa Meno Honrosa Meno Honrosa

EUREKA! N24, 2006

75

Sociedade Brasileira de Matemtica

Nvel Universitrio
NOME Humberto Silva Naves Bernardo Freitas Paulo da Costa Alex Corra Abreu Rafael Daigo Hirama Digo Veloso Ucha Fbio Dias Moreira Lus Daniel Barbosa Coelho Carlos Stein Naves de Brito Yuri Gomes Lima Rafael Marini Silva Murilo Vasconcelos Andrade Thiago Barros Rodrigues Costa Felipe Rodrigues Nogueira de Souza Leonardo Augusto Zo Vitor Gabriel Kleine Estillac Lins Maciel Borges Filho Rodrigo Roque Dias Eduardo de Moraes Rodrigues Poo Gustavo Gomes de Araujo Raphael Constant da Costa Davi Maximo Alexandrino Nogueira Jorge Peixoto de Morais Neto Eduardo Ferraz Castelo Branco Ferreira Eduardo Famini Silva Moyses Afonso Assad Cohen Kellem Corra Santos Evandro Makiyama Thiago da Silva Sobral Pedro Paiva Zhlke Dioliveira Helder Oliveira de Castro Thiago Costa Leite Santos Marcos Francisco Ferreira Martinelli Rogrio de Assis Medeiros Samuel Barbosa Feitosa Elder Rodrigo Barbosa Campos Francisco Bruno de Lima Holanda Giovana Siracusa Gouveia Henrique Roscoe de Oliveira CIDADE ESTADO S. J. dos Campos SP Rio de Janeiro RJ Niteri RJ Campinas SP Teresina PI Rio de Janeiro RJ Rio de Janeiro RJ S.J. dos Campos SP Fortaleza CE Vila Velha ES Macei AL Fortaleza CE So Paulo SP Nilpolis RJ Mogi das Cruzes SP Belm PA So Paulo SP So Paulo SP Ribeiro Preto SP Rio de Janeiro RJ Fortaleza CE Goinia GO Rio de Janeiro RJ Rio de Janeiro RJ Rio de Janeiro RJ Rio de Janeiro RJ So Paulo SP S.J. dos Campos SP Braslia DF Mogi das Cruzes SP So Paulo SP Rio de Janeiro RJ Franco da Rocha SP Fortaleza CE Rio de Janeiro RJ Fortaleza CE Recife PE Braslia DF PRMIO Ouro Ouro Ouro Ouro Ouro Ouro Prata Prata Prata Prata Prata Prata Prata Prata Prata Bronze Bronze Bronze Bronze Bronze Bronze Bronze Bronze Bronze Bronze Meno Honrosa Meno Honrosa Meno Honrosa Meno Honrosa Meno Honrosa Meno Honrosa Meno Honrosa Meno Honrosa Meno Honrosa Meno Honrosa Meno Honrosa Meno Honrosa Meno Honrosa

EUREKA! N24, 2006

76

Sociedade Brasileira de Matemtica

AGENDA OLMPICA
XXVIII OLIMPADA BRASILEIRA DE MATEMTICA NVEIS 1, 2 e 3 Primeira Fase Sbado, 10 de junho de 2006 Segunda Fase Sbado, 2 de setembro de 2006 Terceira Fase Sbado, 28 de outubro de 2006 (nveis 1, 2 e 3) Domingo, 29 de outubro de 2006 (nveis 2 e 3 - segundo dia de prova). NVEL UNIVERSITRIO Primeira Fase Sbado, 2 de setembro de 2006 Segunda Fase Sbado, 28 e Domingo, 29 de outubro de 2006

XII OLIMPADA DE MAIO 13 de maio de 2006

XVII OLIMPADA DE MATEMTICA DO CONE SUL 5 a 11 de maio de 2006 Escobar, Argentina

XLVII OLIMPADA INTERNACIONAL DE MATEMTICA 8 a 19 de julho de 2006 Ljubljana - Eslovnia.

XIII OLIMPADA INTERNACIONAL DE MATEMTICA UNIVERSITRIA 20 a 26 de julho de 2006 Odessa, Ucrnia

XXI OLIMPADA IBEROAMERICANA DE MATEMTICA 22 de setembro a 01 de outubro de 2006 Equador

IX OLIMPADA IBEROAMERICANA DE MATEMTICA UNIVERSITRIA 18 de novembro de 2006

EUREKA! N24, 2006

77

Sociedade Brasileira de Matemtica

COORDENADORES REGIONAIS
Alberto Hassen Raad Amrico Lpez Glvez Amarsio da Silva Arajo Andreia Goldani Antonio Carlos Nogueira Ali Tahzibi Benedito Tadeu Vasconcelos Freire Carlos Alexandre Ribeiro Martins Carlos Frederico Borges Palmeira Claus Haetinger Cleonor Crescncio das Neves Cludio de Lima Vidal Edson Roberto Abe lio Mega der Luiz Pereira de Andrade Eudes Antonio da Costa Florncio Ferreira Guimares Filho Ivanilde Fernandes Saad Janice T. Reichert Joo Bencio de Melo Neto Joo Francisco Melo Libonati Jorge Costa Duarte Filho Jos Cloves Saraiva Jos Luiz Rosas Pinho Jos Vieira Alves Jos William Costa Krerley Oliveira Licio Hernandes Bezerra Luzinalva Miranda de Amorim Mrio Rocha Retamoso Marcelo Rufino de Oliveira Marcelo Mendes Newman Simes Ral Cintra de Negreiros Ribeiro Ronaldo Alves Garcia Rogrio da Silva Igncio Reginaldo de Lima Pereira Reinaldo Gen Ichiro Arakaki Ricardo Amorim Srgio Cludio Ramos Seme Guevara Neto Tadeu Ferreira Gomes Toms Menndez Rodrigues Turbio Jos Gomes dos Santos Valdenberg Arajo da Silva Valdeni Soliani Franco Vnia Cristina Silva Rodrigues Wagner Pereira Lopes (UFJF) (USP) (UFV) FACOS (UFU) (USP) (UFRN) (Univ. Tec. Fed. De Paran) (PUC-Rio) (UNIVATES) (UTAM) (UNESP) (Colgio Objetivo de Campinas) (Colgio Etapa) (UNESPAR/FECILCAM) (Univ. do Tocantins) (UFES) (UC. Dom Bosco) (UNOCHAPEC) (UFPI) (Grupo Educacional Ideal) (UFPB) (UFMA) (UFSC) (UFPB) (Instituto Pueri Domus) (UFAL) (UFSC) (UFBA) (UFRG) (Grupo Educacional Ideal) (Colgio Farias Brito, Pr-vestibular) (Cursinho CLQ Objetivo) (Colgio Anglo) (UFGO) (Col. Aplic. da UFPE) (Escola Tcnica Federal de Roraima) (LAC - Laboratrio Associado de Computao) (Centro Educacional Logos) (IM-UFRGS) (UFMG) (UEBA) (U. Federal de Rondnia) (UFPB) (U. Federal de Sergipe) (U. Estadual de Maring) (U. Metodista de SP) (CEFET GO) Juiz de Fora MG Ribeiro Preto SP Viosa MG Osrio RS Uberlndia MG So Carlos SP Natal RN pato Branco - PR Rio de Janeiro RJ Lajeado RS Manaus AM S.J. do Rio Preto SP Campinas SP So Paulo SP Campo Mouro PR Arraias TO Vitria ES Campo Grande MS Chapec SC Teresina PI Belm PA Joo Pessoa - PB So Luis MA Florianpolis SC Campina Grande PB Santo Andr SP Macei AL Florianpolis SC Salvador BA Rio Grande RS Belm PA Fortaleza CE Piracicaba SP Atibaia SP Goinia GO Recife PE Boa Vista RR SJ dos Campos SP Nova Iguau RJ Porto Alegre RS Belo Horizonte MG Juazeiro BA Porto Velho RO Joo Pessoa PB So Cristovo SE Maring PR S.B. do Campo SP Jata GO

EUREKA! N24, 2006

78

Potrebbero piacerti anche